Наука


Ответить в тред Ответить в тред

Check this out!
<<
Назад | Вниз | Каталог | Обновить тред | Автообновление
566 45 134

Тред тупых вопросов и умных ответов. Прошлый >>486528 (OP) Тред тупых вопросов 22/12/19 Вск 19:37:03 4893211
image.png (105Кб, 277x182)
277x182
Тред тупых вопросов и умных ответов.

Прошлый >>486528 (OP)
Аноним 22/12/19 Вск 19:39:41 4893232
Шо такое атомный спектор????????
Аноним 22/12/19 Вск 19:54:39 4893263
15685312952670.png (522Кб, 754x501)
754x501
Аноним 22/12/19 Вск 19:58:13 4893274
image.png (651Кб, 700x406)
700x406
Поясните космический эффект этой картинки
Аноним 22/12/19 Вск 19:58:48 4893285
>>489323
Разложение говен на компоненты говености.
Аноним 22/12/19 Вск 20:03:47 4893296
>>489327
В том, что на картинке буквальное интерпретирование математических терминов.
22/12/19 Вск 21:07:49 4893337
>>489321 (OP)
я уж думал попрыгун помер, дебильный прикол.
Аноним 22/12/19 Вск 21:23:24 4893348
>>489328
ничего не понял можно подробние?
Аноним 22/12/19 Вск 22:56:04 4893419
>>489333
Он тебя переживёт.
Аноним 22/12/19 Вск 23:03:26 48934310
Так, а схуяли собственно энштейн говорил что движение относительно, когда он сам же блять вывел эту свою формулку е=mc^2??
Аноним 22/12/19 Вск 23:09:47 48934511
>>489343
Как движение связано с твоей формулой? Ты ебанутый?
Аноним 22/12/19 Вск 23:22:20 48934712
>>489345
Ты не очень умный?
Чем быстрее тело движется тем труднее его разгонять, движущееся тело труднее двигать и разгонять, значит движение абсолютно а не относительно как пиздел эйнштейн.

Хотя он сам же и вывел этот закон, сам вывел и сам пиздел что относительно, бред/лол
Аноним 22/12/19 Вск 23:58:11 48935013
>>489347
Проиграл с твоей "логики". Про системы отсчета ты видимо забыл. Если я буду двигаться рядом с телом, которое тебе очень сложно разгонять, то мне его в моей системе отсчета разгонять будет так же просто, будто оно покоилось.
Аноним 23/12/19 Пнд 00:22:07 48935214
>>489350
Пиздёж.

Мне кажется если буду лететь в корабле со значительной частью скорости света то мне будет не как обычно двигать предметы находящиеся рядом со мной.
Аноним 23/12/19 Пнд 00:32:44 48935415
>>489352
>Мне кажется
В церковь сходи.
Аноним 23/12/19 Пнд 00:54:02 48935516
>>489354
А ты обладаешь точными знаниями по этому вопросу? Что-то мне подсказывает, что нет, иначе бы ты ответил, а не натужно петросянил.
Аноним 23/12/19 Пнд 01:05:18 48935617
>>489355
>а что, тебе слабо?
Серьезно? Может тебе еще тебе лекцию прочитать? За это деньги вообще-то платят. Я сегодня четыре пары провел, ты думаешь мне хочется сейчас какому-то ленивому опездалу на дваче в час ночи что-то объяснять. Тема то простая, открой учебник, да посмотри о чём там речь. Возьми три формулы, формулу связывающую энергию и импульс тела. Формулу сложения скоростей. И формулу сокращения длинн. И просто на листочке составь маленькую задачку, где ты в корабле толкаешь мячик, а твой друг наблюдает за этим со стороны. Проверь, сходятся ли наблюдения друга и твои.
Аноним 23/12/19 Пнд 01:18:32 48935718
>>489356
>Серьезно?
Почему нет?
>Может тебе еще тебе лекцию прочитать?
А ты сможешь?
Аноним 23/12/19 Пнд 01:49:48 48936019
>>489343
Все верно, эта формулка к движению отношения и не имеет.
Аноним 23/12/19 Пнд 02:08:46 48936220
>>489343
>когда он сам же блять вывел эту свою формулку е=mc^2??

Вообще-то он вывел вот это, именно за это он стал тем, кем стал

Rμν - 1/2 ⋅ gμν ⋅ R + Λgμν = 8πG/c4 ⋅ Tμν

А то что ты написал это всего лишь о связи энергии покоя тела и его массы покоя.
Аноним 23/12/19 Пнд 02:09:03 48936321
>>489356
>Серьезно? Может тебе еще тебе лекцию прочитать? За это деньги вообще-то платят
Ааааэахааэ, тупая пизда-вниманиеблядь
Если ты ещё и с отростком между ног, то это вообще джекпот.
Аноним 23/12/19 Пнд 02:39:57 48936522
>>489362
Ну не важно что он там именно вывел, важно что он кроме всего прочего вывел то, что для разгона тела имеющего массу до скорости света понадобится бесконечное количество энергии.

Т.е. уже движущееся тело разгонять сложнее.
Аноним 23/12/19 Пнд 03:19:41 48936923
>>489365
>Ну не важно что он там именно вывел
Как это не важно? Это общепринятая теория гравитация связывающая метрику искривлённого пространства-времени со свойствами заполняющей его материи.

>Т.е. уже движущееся тело разгонять сложнее.

А что ты понимаешь пол "движущиеся телом"?

Аноним 23/12/19 Пнд 03:20:04 48937024
>>489369
>под "движущимся телом"?
фикс
Аноним 23/12/19 Пнд 11:46:06 48940425
>>489321 (OP)
Так, а схуяли собственно решили что сингулярность в центре вращающейся чд не точка/сфера, а тороид-бублик?
Аноним 23/12/19 Пнд 12:38:07 48940726
Вопрос химикам о быте. Из чего сделаны пластиковые бутылки для подсолнечного масла ? Можно ли масло хранить в таре из полипропилена или полиэтилена ? На сколько вредно или совсем безвредно если не нагревать ?
Аноним 23/12/19 Пнд 12:59:28 48941227
>>489407
Очевидно сделаны из пластика ( PET - Полиэтилентерефталат).
Да можно, только убедись, что тара пищевая, а не от химреактивов или прочего непищевого говна.
Большинство пластиков довольно химически инертны и не растворяются в пищевых продуктах, поэтому и используют как пищевую тару. Даже если и сожрешь нанопластик, то он выйдет с говном. Главное только на солнце не ставь.
Аноним 23/12/19 Пнд 13:08:43 48941528
>>489404
>Так, а схуяли собственно решили что сингулярность в центре
Это всего лишь гипотеза, так что ты можешь смело предложить свою гипотезу, со своей сингулярностью. Только не забудь разработать математический аппарат для своей гипотезы.
Аноним 23/12/19 Пнд 13:11:39 48941629
>>489415
Я про это и спросил схуяли у них возникла гипотеза что там бублик.

Понятно что они это проверить не могут.
Аноним 23/12/19 Пнд 14:19:04 48942230
>>489416
>Я про это и спросил схуяли у них возникла гипотеза что там бублик.
Эту ахинею с сингулярностями астрофизикам подкинули математики и астрофизики на нее клюнули. Когда математик не может свои уравнения решить, то он все валит на математическую сингулярность, дескать тут ничего решить нельзя, возникает ебаная сингулярность.
Экспериментально создать черную дыру невозможно, наблюдений ее тоже нет, так что остается строить гипотезы, некоторые из которых откровенно псевдонаучные, такие как наличие некой "сингулярности" внутри черной дыры.

Есть модели черных дыр без сингулярностей внутрях. Луркай.
Аноним 23/12/19 Пнд 14:35:00 48942531
>>489422
Да блять причём тут сингулярность, вопрос про модели вращающихся чд.
Аноним 23/12/19 Пнд 14:51:57 48942732
Стоит упомянуть Эйнштейна и скорость света, особенно строение света - и начинается срач на сотни сообщений, пока кто нибудь не вспомнит о кварках. Тогда срач сворачивается в точку, как и любая кварковая модель.
Аноним 23/12/19 Пнд 14:58:34 48943033
>>489425
>вопрос про модели вращающихся чд.
Ответ в самом вопросе. Это одна из моделей. Есть модель не вращающейся черной дыры. Есть модель гравастара, есть модель звезды темной энергии, выбирай любую на свой вкус. У нас нет никаких данных о тех объектах, что мы обсуждаем, известно лишь, что они не излучают в электромагнитном спектре, но у них есть джеты, что они очень очень массивны и искажают пространство вызывая наблюдаемый нами эффект гравитационного линзирования Вот и все что мы знаем.
Аноним 23/12/19 Пнд 14:59:30 48943134
>>489427
>Тогда срач сворачивается в точку
Это если не вспомнят о глюонах.
Аноним 23/12/19 Пнд 15:17:02 48943535
>>489430
Бля, хватит писать чтобы писать.
Аноним 23/12/19 Пнд 15:19:05 48943736
Аноним 23/12/19 Пнд 15:19:48 48943937
>>489437
Тебе не хватает внимания и общения.
Аноним 23/12/19 Пнд 15:30:51 48944138
>>489439
А твоей маме похоже не хватало йода когда она была беремена тобой.

Твой вопрос

>>489404
>Так, а схуяли собственно решили что сингулярность в центре вращающейся чд не точка/сфера, а тороид-бублик?

Через два поста твоя реплика:

>>489425
>Да блять причём тут сингулярность
Аноним 23/12/19 Пнд 21:31:58 48947339
>>489407
Не храни машинное масло в полиэтиленовых пакетах, оно просачивается наружу
Аноним 23/12/19 Пнд 22:30:15 48947740
Поясните про разброс в размерах схожих видов в природе. Самый яркий пример с кошачьими, есть львы и тигры, а есть черноногие кошки, которые не больше домашней кошки. То есть у них был один предок, но что заставило пойти по таким разным путям развития. У собак вроде такого не наблюдается, волки, шакалы и лисы примерно одного размера.
Аноним 23/12/19 Пнд 23:32:10 48947941
Может кто-нибудь на пальцах объяснить парадокс, который возникнет если передать информацию быстрее скорости света?
Аноним 23/12/19 Пнд 23:34:43 48948042
>>489477
Грубо говоря обе версии оказались выйгрышными.
Аноним 24/12/19 Втр 00:28:12 48948343
>>489479
Ребята не стоит вскрывать эту тему. Вы молодые, шутливые, вам все легко. Это не то. Это не электродинамика и даже не парадокс еблана. Сюда лучше не лезть. Серьезно, любой из вас будет жалеть. Лучше закройте тему и забудьте что тут писалось.
Аноним 24/12/19 Втр 00:56:23 48948444
>>489477
>У собак вроде такого не наблюдается, волки, шакалы и лисы примерно одного размера.
У ныне живущих да, но были твари и крупнее.

https://en.wikipedia.org/wiki/Dire_wolf
Аноним 24/12/19 Втр 02:11:18 48948545
image.png (79Кб, 1024x640)
1024x640
image.png (370Кб, 735x507)
735x507
image.png (1124Кб, 765x525)
765x525
>>489484
Ужасный волк был примерно того же размера, что и самые крупные современные серые волки, весил в среднем 60 кг


МОСБАХСКИЙ ЛЕВ Длина тела взрослой особи достигала 2,5 метра без учёта хвоста, в высоту львы были около 1,3 метра. Весил мосбахский лев до 450 кг.

Ксеносмилус весил до 350 кг, а размер тела около 2 метров.
Аноним 24/12/19 Втр 02:21:44 48948646
hpiaTzj.png (448Кб, 1224x429)
1224x429
>>489485
А ты в курсе, что самый крупный за всю историю Земли млекопитающий хищник был... копытным?

Эндрюсарх был самым крупным известным сухопутным хищным млекопитающим всех времён.

Длина черепа составляла 83,4 см (у современного бурого медведя — не более 55 см), а ширина черепа на уровне скуловых дуг — 56 см. На основании реконструкций предполагается, что общая длина тела эндрюсарха составляла 3,8 м; по другим оценкам, длина тела составляла 4,5 м, при длине хвоста до 1,5 м. Высота в холке, видимо, составляла до 1,6 м. Вес мог достигать одной тонны.

https://ru.wikipedia.org/wiki/%D0%AD%D0%BD%D0%B4%D1%80%D1%8E%D1%81%D0%B0%D1%80%D1%85

Аноним 24/12/19 Втр 03:52:36 48949247
>>489486
Это конечно все интересно, но речь шла о котиках. Олсо, утверждать самый крупный мы не можем, как и не можем даже назвать цифру сколько вымерших видов нам неизвестно, сколько не открыто.
Аноним 24/12/19 Втр 04:07:32 48949348
>>489492
> но речь шла о котиках
не только
Аноним 24/12/19 Втр 04:10:53 48949449
>>489493
И копыт на картинке я не увидел. Какой-то крысо-бегемот переросток.
Аноним 24/12/19 Втр 04:11:58 48949550
>>489479
Никаких парадоксов нет.
Аноним 24/12/19 Втр 04:14:17 48949651
>>489494
>И копыт на картинке я не увидел.
чет кекнул мальца
Аноним 24/12/19 Втр 04:37:22 48949852
15565657283050s.jpg (2Кб, 170x112)
170x112
>>489496
>чет кекнул мальца
Аноним 24/12/19 Втр 04:59:48 48949953
>>489498
Ты смешной челик
Аноним 24/12/19 Втр 05:46:12 48950254
Портит ли литий ионные и литий полимерные аккумуляторы глубокий разряд и длительно(годами) отсутствие эксплуатации? Если да - из за чего?
Что такое магнитное поле если поле математический объект? Аноним 24/12/19 Втр 11:41:29 48954155
Да прибудет срач.
Аноним 24/12/19 Втр 16:31:40 48958956
>>489541
Что такое расстояние, если пространство это математический объект?
Аноним 24/12/19 Втр 16:39:41 48959057
>>489343
Нет такой формулы. Есть Eo=mc² или E=m(rel)c². И она не имеет отношения к движению. То, что электромагнитные законы соблюдаются во всех инерциальных системах отсчёта (а об этом СТО, следствием которой является эта формула) - эмпирический факт, который Эйнштейн сумел облечь в теорию и обосновать.
>>489479
Это не парадокс, это факт.
Аноним 24/12/19 Втр 16:40:10 48959158
Что используется как ориентир для определения альтитуды на планетах, где нет воды?
Аноним 24/12/19 Втр 16:42:34 48959259
>>489541
Поле - явление. Математический объект, это тензорное пространство, которое его описывает.
Аноним 24/12/19 Втр 16:46:13 48959360
>>489541
Да хз почему ещё не придумали как измерять поля
Аноним 24/12/19 Втр 16:48:56 48959561
>>489593
Бля лолка, все измерения это измерения полей.
Что ещё измерять?
Аноним 24/12/19 Втр 16:49:20 48959662
>>489595
И сколько весит поле?
Аноним 24/12/19 Втр 16:49:28 48959763
>>489589
Если поле - не поле, но явление, то поле это способ описать явление. Способ не физичный, в отличии от расстояния. Потому что поле - это поле.
>>489592
this
>>489593
Ты спроси лучше, почему существуют плюс и минус. Это же просто факт, но не модель зарядов. В некоторых учебниках пишут о поле как об особой материи. Мне думается, это уход от объяснений.
Аноним 24/12/19 Втр 16:50:31 48959864
>>489596
>весит
Веса нет, это лишь твоя ограниченность.
Аноним 24/12/19 Втр 16:50:56 48959965
mg-400-1.jpg (466Кб, 1600x1200)
1600x1200
>>489593
Чобля!? А мужики то не знали.
Аноним 24/12/19 Втр 16:54:37 48960066
>>489597
Поле это информация какая-то вроде
Аноним 24/12/19 Втр 16:55:03 48960167
Аноним 24/12/19 Втр 16:55:20 48960268
>>489599
И что они измеряют?
Аноним 24/12/19 Втр 17:00:23 48960369
Аноним 24/12/19 Втр 23:49:32 48964870
>>489601
>длину
Опять это лишь твоя ограниченность.
Аноним 25/12/19 Срд 01:17:53 48966171
>>489479
Существующая теория начинает возвращать очевидно хуевые ответы на бумажке, создать новую и проверить все на практике нельзя потому что не придумали как разгоняться быстрее скорости света без спидов и психоделиков.
Аноним 25/12/19 Срд 04:11:21 48968172
>>489661
За сверхсветом отправляйтесь в 20 век.
Аноним 25/12/19 Срд 06:12:40 48969573
>>489648
Зря ты так на него.
Хоть и тралет
Аноним 25/12/19 Срд 06:13:59 48969674
image.png (498Кб, 1379x599)
1379x599
У меня одного баг? Скролю страницу вниз, дохожу до до второй и как бы перескакиваю через вторую страницу на третью, 20 тредов не отображаются, но их можно найти в каталоге
Аноним 25/12/19 Срд 09:01:12 48972875
>>489591
Карту высот на планетах составляли через радиолокацию. Дополнительно исследуются давление атмосферы.
После обработки данных выбирают среднее на планете, это и будет нулем.
Многие спутники обладают "морями" как Луне. Т.е обширные участки с маленькими перепадами высот. Их часто используются за нулевую отметку.
Или иногда не парятся выбирают какой-нибудь выделяющуюся деталь ландшафта и считают ее нулем высоты.
Аноним 25/12/19 Срд 09:42:36 48973376
А чё, реально антарктида из-за массы льдов утопилась в мантии, а раньше горы на ней были чуть не с гималаи высотой над уровнем моря?
Аноним 25/12/19 Срд 09:51:00 48973577
y0weqdTvIg.jpg (91Кб, 604x463)
604x463
Аноним 25/12/19 Срд 10:35:29 48974678
>>489733
Есть такой эффект, но проседает аж целая плита. Горы на земле высокими не могут быть, поскольку слишком высокая эрозия. Впрочем Антарктика в своей молодости была постоянно сжата со всех сторон, поэтому она приехала на полюс. Горы это остаточное явление от переезда. Больше напоминает Африку, только без мантийных плюмов. Гимали отдельный разговор, поскольку там маленькая плита врезается в большую, которая является спайкой из нескольких.
Аноним 25/12/19 Срд 12:47:19 48975579
15763725180540.mp4 (5607Кб, 330x480, 00:01:05)
330x480
Это че получается, нам все время врали, что у котов нет самосознания?
Аноним 25/12/19 Срд 12:55:36 48975680
>>489755
>самосознания
Нет, не врали, просто у тебя его тоже нет.
Аноним 25/12/19 Срд 13:44:53 48975981
Аноним 25/12/19 Срд 13:52:26 48976182
>>489759
Потому что твой состав из элементарных частиц, плюс изначальные параметры системы, плюс возможно истинно случайные квантовые эффекты.
Из всего этого "ты" ну никак не складывается.
А больше ничего и нет.

А если и есть, я рад послушать как из этого чего-то по-твоему получаешься "ты".
Аноним 25/12/19 Срд 13:58:06 48976283
>>489761
Ну я в зеркале могу себя узнать, коты с вебм тоже могут, а говорили, что они не могут твердо и четко и это только человек и ещё пара зверей такие умные.
Аноним 25/12/19 Срд 13:59:54 48976384
>>489762
Ну да, события происходят разные.
Одни взаимодействия вызывают другие, это нормально. Это везде так работает одинаково.
Аноним 25/12/19 Срд 14:33:17 48976985
>>489761
Сознание это способность интегрировать разнородные сигналы и концепты (в лингвистическом значении) речью (в т.ч. внутренней) от первого лица. Мозг (здоровый) обладает этой интегративностью и способен ее репрезентовать. Элиминативисты идут нахуй. Как только научатся говорить без первого лица единственного числа, тогда и поговорим. Пусть докажут, что это возможно. Я утверждаю, что нет, и не обязан доказывать своё отрицание.
Аноним 25/12/19 Срд 16:09:03 48978786
Аноним 25/12/19 Срд 22:52:50 48981787
Аноним 26/12/19 Чтв 01:38:22 48982888
Поясните за эксперимент с двумя щелями.
Если поставить детектор частиц в щелях, но не смотреть в приборы, будет интерференционный рисунок, или он пропадает, как только ставится детектор?
Аноним 26/12/19 Чтв 01:39:40 48982989
>>489828
>он пропадает, как только ставится детектор
Это. Но ты должен его еще включить.
Аноним 26/12/19 Чтв 04:20:17 48984490
>>489828
интерференционный эффект пропадает, как только мы воздействуем на квантовую систему классической большой. смысл не в наблюдателе и магии там никакой нет. посмотри лекции Эмиля Ахметова, он просто и серьёзно объясняет, смывая к чертовой матери всю мистику
Аноним 26/12/19 Чтв 04:28:26 48984591
S91226-04251394.jpg (401Кб, 1080x2160)
1080x2160
объясните, пожалуйста, за счет каких эффектов свет от ламп/вспышки образует такую картину.
Аноним 26/12/19 Чтв 04:34:54 48984792
Почему при высокой температуре возникает синдром Алисы в стране чудес? Почему мне до сих пор становится не по себе, а в руках снова появляется предательская лёгкость, как тогда, когда я был ребенком, если я представляю себе крохотный ключ весом в сто тонн, или огромный чугунный шар, весящий как пушинка?
Аноним 26/12/19 Чтв 13:21:22 48991693
ffOaSBjt400x400.jpg (20Кб, 337x337)
337x337
>>489755
>видосики, где за кадром колбасой машут, выдавать за пруф самосознания.
Аноним 26/12/19 Чтв 13:24:34 48991794
Аноним 26/12/19 Чтв 18:56:17 48995795
у меня есть 100 карточек. они делятся по двум признакам, вероятность каждого 5%, в том числе оба признака могут быть у одной и той же карточки. какова вероятность в %, что у карточки будут оба признака? а сколько карточек вероятнее всего таких будет? а если карточек 200?
я тупой не пинайти
Аноним 26/12/19 Чтв 19:15:57 48996296
>>489916
Двачую, это все китайский заговор.
26/12/19 Чтв 19:24:26 48996397
>>489957
Это же типовая задача, пиздуй учебник читать или хотя бы лекцию на ютубе посмотри.
Аноним 26/12/19 Чтв 19:31:32 48996498
>>489963
я не могу понять, как эти 5% и 5% завероятнить
Аноним 26/12/19 Чтв 19:37:13 48996599
26/12/19 Чтв 19:40:25 489966100
>>489964
Ебать ты мудак
Если независимые состояния, то вероятности перемножаются. Для твоей сраной карточки будет 0,25% вероятности совпадения.
В случае с множеством попыток, то там считаем через биномиальные коэффициенты. Это блядь проходят на первых лекциях. Там уж сам считай.
Аноним 26/12/19 Чтв 19:48:29 489967101
>>489966
Я это даже в школе проходил. И перепроходил в медвузе аж два раза, лол (изумительная согласованность программы, лол).
Аноним 26/12/19 Чтв 20:09:20 489969102
>>489965
пасиба
>>489966
да я хлебушек какие лекции лол
Аноним 26/12/19 Чтв 20:35:05 489971103
>>489321 (OP)
Посоветуйте учебники по нейронным сетям мозга.
Аноним 27/12/19 Птн 17:31:33 490031104
Аноны блядждж почему вокруг пламени свечи возникает чуть чуть вытянутая радужная сфера??? В темноте.
Аноним 27/12/19 Птн 19:31:29 490040105

Вот у нас есть генератор - ну там обмотка, ротор, статор. Эта ебала крутится и магнитным полем двигает электроны, да? Поток электронов это же ток? Но у нас есть закон сохранения материи - электроны не могут появляться просто так, значит магнитное поле как бы из метала выталкивает электроны, значит они когда-нибудь закончатся и генератор не сможет выдавать ток? Получится метал останется без электронов и разрушится?
Аноним 27/12/19 Птн 19:32:25 490041106
>>489845
за счет линз в объективе
Аноним 27/12/19 Птн 19:33:59 490042107
>>489755
>нам все время врали, что у котов нет самосознания?
а кто сказал что у них его нет, дебс? если ты насчет самоидентификации в зеркале, то это другое и она есть у котов которые регулярно втыкают в зеркало, просто мало кому из котов это интересно туда втыкать
Аноним 27/12/19 Птн 19:39:47 490043108
>>490040
Нет, это же цепь. Статор >> провода >> лампа >> провода >> статор.
Аноним 27/12/19 Птн 19:44:21 490044109
>>490040
И да, например при фотоэффекте металлы могут деградировать как ты описал, но в этом случае система открыта: свет >> металл1 >> электрон - выброс >> металл2
Аноним 27/12/19 Птн 21:09:57 490047110
>>490040
Ток это свойство ЭМ поля или ток это как ЭМ поле реагирует на движение заряженных частиц.
В проводниках, да и вообще в любом твердом теле, движутся не сами электроны, а их состояния в кристаллической решетке. ЭМ похуй на такие мелочи, оно вообще не различает электроны от друг друга. Тут уместна аналогия с цепочным падение доминошек. Каждая домишка весьма ограничена в движении, но на большом масштабе мы видим движения множества доминошек целиком как большую волну.
Аноним 27/12/19 Птн 22:35:52 490049111
>>489321 (OP)
Силовые линии магнитного поля называют мнимыми линиями.
И рисунок из опилок на бумаге рядом с магнитом можно объяснить тем что опилкам "энергетически выгоднее" слипаться в подобие линий. Если где-то опилок больше выстрено в ряд, в эжтом месте сильнее поле и с больше йвероятностью опилка рядом притянется к формации.
Но посмотрите на магнитные жидкости! Там рисунок из шипов.
https://www.youtube.com/watch?v=OjYrSAW3QQg

Разве может там быть такой же механизм как у опилок?
А может магнитное поле всёже неоднородно и аналоги "силовых линий" существуют в еральности?
Аноним 27/12/19 Птн 22:48:11 490052112
>>490049
Магнитное поле такая же абстракция как и силовые линии.
С жидкостью все совершенно то же, что и с опилками. Эта жидкость не жидкость, а взвесь тех же опилок.
Аноним 27/12/19 Птн 23:48:28 490054113
>>490040
Там электроны по кругу крутятся. Считай как водяное колесо, только из электронов.
Аноним 28/12/19 Суб 09:13:39 490092114
>>490049
Силовые линии это в первую очередь абстракция для визуализации. Само магнитное поле непрерывно.
Опилки на бумаги выстраиваются в линию, потому что они имеют конечный размер и они взаимодействуют сами с собой. В итоге они по бокам себя отталкиваются, а вдоль притягиваются. Так и получаются "пучок" линий.
С жидкостями отдельный разговор. У них есть параметр свободная энергия определяемый поверхностным натяжений. Энергия этого натяжений определяется площадью и поверхностным натяжением.
Когда на поверхности жидкости магнитное поле начинает превосходить по силе поверхностное натяжение, нужно как то сохранить равновесие, т.е надо увеличить свободную энергию. Сила поверхностного натяжения постоянна, а вот площадь нет. И жидкость увеличивает свободную энергию за счет увеличение площади(через сферические гармоники или создание сингулярностей). Поскольку взаимодействие магнитного поля зависит от ориентации к поверхности, можно увеличить площадь поверхности, но минимизировать взаимодействие с полей. Вот так формируются шипы и иголочки, которые имеют большую площадь, то минимальное взаимодействие с магнитным полем.
Аноним 28/12/19 Суб 15:45:30 490160115
Аноним 28/12/19 Суб 15:54:06 490161116
Почему фашисты так любят приводить в пример биологию в ответ на ЛГБТ, гендеры и прочее сжв?
Вообще биология эта хуйня раз ей так вертеть можно? Ее всякого рода научпоперы с ютуба обожают. Одни ей оправдывают то что ГЕНДЫРА ДВА ПАНЯТНА, другие А ЖЫВОТНЫЕ ВОТ ТОЖЕ ЕБАЛИСЬ С ДРУГСДРУГОМ У НИХ ТОЖЕ КОШКИ ЛЕСБИЯНКИ ЕСТЬ Как проститутка.
Аноним 28/12/19 Суб 15:55:54 490162117
>>490161
>Вообще биология эта хуйня
А чо у тебя по биологии в прошлой четверти было?
Аноним 28/12/19 Суб 16:04:13 490163118
>>490162
Не окончил 9 классов и сбежал из школы в 7-ом из-за травли белых цис мразей, у которых было оба родителя, машины, компьютеры, новые консоли, хорошая одежда и обувь.
Аноним 28/12/19 Суб 16:05:51 490164119
В школе так же не давали научных знаний, 95% - это пропаганда и промывка мозгов. Лучше бы в 3-4 бросил, все равно кроме чтения ничего и не пригодилось в жизни.
Аноним 28/12/19 Суб 16:53:03 490169120
сжв фашики не нужны
Аноним 28/12/19 Суб 16:58:06 490172121
>>490163
>сбежал из школы в 7-ом
Куда подался?
Аноним 28/12/19 Суб 17:41:25 490177122
>>490161
При чем тут биология? Это проблема идеологии, а не биологии. Прикинь, если захотеть, то и физику можно поделить на арийскую и еврейскую, и генетику объявить буржуазной лженаукой.
Аноним 28/12/19 Суб 17:55:45 490180123
>>490177
Значит современной биологии доверять нельзя. Некоторые на серьезных щах вещают о зависимости между генетикой и политическими взглядами, я имею ввиду либерализм/тоталитаризм и т.п. противоположности, что интересно люди не понимают это как совокупность всех факторов а понимают как нечто предопределенное и стоящее за гранью их сознания, отчего получаем глубокое убеждение и ебланов. Вот казалось бы, да - зависимость есть, но очевидно есть и другие высшие зависимости, но нет, об этом эти некоторые не говорят, точнее ставят акцент на генах.
Это как человека прожившего всю жизнь в лесу объявить генетическим уродом, смешно же блядь.
Аноним 28/12/19 Суб 23:17:45 490228124
>>490164
Из советского союза капчуешь?
Аноним 28/12/19 Суб 23:51:34 490236125
>>490180
Так это это не проблемы науки, снова. Если ты начнёшь вдумчиво читать источники и общаться со специалистами, то быстро поймёшь, что нет ни у кого, кто действительно сечет в методологии, иллюзий относительно корреляций. То, что ты бомбишь от подмен корреляционных связей на детерминированные, значит, что ты получаешь сведения от новучпоперсов, или, в лучшем случае, не заглядываешь дальше абстрактов, не говоря о том, чтобы потеребить исследованию приложения, куда, обычно, подводные камни прячут из-за ебанутейшей системы переодевания своих хиршей, но это снова отдельная проблема, и те, кому надо, пусть и с затратами лишнего времени и еблей мамок хитрожопых исследователей, умеют от этого предохраняться. Короче, проблемы не у биологии, в у хуманов в головах. Биология просто ближе по уровню абстракции к обыденности, у нее нет проблем (а, точнее, охуительных антипрофанных фич) с интуитивностью, как у физики. Она, в отличие от физики, с конями работает без вакуума и не округляет, что увеличивает степень погрешности у моделей. Это делает ее самой доступной жертвой профанации и корыстных делишек.
Аноним 28/12/19 Суб 23:52:54 490238126
>>490236
>передергивания своих хиршей*
Аноним 29/12/19 Вск 20:46:23 490407127
Как Китай монополизировал рынок редких земель? Неужели только на их территории руды?
Аноним 29/12/19 Вск 21:43:00 490409128
>>490407
Дело в не наличие руд, а в их обогащении и переработке. Редкоземельные элементы в природе находятся в рассеянном виде, и вдобавок их они имеют тенденцию к изоморфному замещение других элементов.
Чтоб выделить лантаноиды нужнен ебунячий по сложности процесс. Китайцы умудрились удешевить и упросить, а потом тупо занялись демпингом цен. Ну и вбухали огромные инвестиции в технологии переработки, да еще наложили анальные патентные ограничения.
В рашке пытались такой финт провернуть с алюминием, но соснули.
Аноним 29/12/19 Вск 21:51:42 490410129
Аноним 29/12/19 Вск 23:14:35 490415130
>>489321 (OP)
Почему, после выдёргивания результатов высокоточного моделирования,
неизбежно усугубляется формирование этого результата в процессе фиксаций к продавливанию его становления?
Что это даёт выблядкам, творящим сие? Нихуя ведь, верно?
Аноним 30/12/19 Пнд 06:43:36 490449131
>>489321 (OP)
Акриламид в пище и воде - причина смерти людей вообще?
Аноним 30/12/19 Пнд 08:35:21 490457132
>>490415
По русскому языку твёрдая тройка?
Аноним 30/12/19 Пнд 11:10:13 490484133
>>490457
Это же шизик-кун, он всегда так пишет.
Аноним 30/12/19 Пнд 13:01:27 490518134
>>490409
Но почему китайцев не послать с их патентами, покуда они сами их не соблюдают (и правильно делают)? Это же стратегическая хуйня. Неужели у США нет денег въебать в разработку и послать китайцев лесом, как они сделали со сланцами и традиционным нефтепроизводителями?
Аноним 30/12/19 Пнд 13:15:36 490522135
>>490518
Ты увидел что китайцы ебашат, но не увидел что делают они это ДЁШЕВО
Нахера их демпенговать, конкурировать и самому это делать, читай внимательнее.

А, ну и плюс пиздецки загрязнять свою экологию.
Аноним 30/12/19 Пнд 20:07:20 490565136
>>489321 (OP)
Почему холодная вода вкуснее?
Аноним 30/12/19 Пнд 23:58:30 490580137
>>490522
Потому, что монополизм чреват. Поставщиков надо дивесифицировать, даже ценой небольших убытков. Это инвестиция в стабильность производственных цепочек. Тем более на эти ресурсы не только гражданская промышленность увязана.
Аноним 31/12/19 Втр 00:50:30 490582138
Аноним 31/12/19 Втр 08:15:14 490605139
>>490565
Вода это безвкусная жидкость, поехавший. Другое дело она может быть приятней в жару, по понятной причине.
Аноним 31/12/19 Втр 08:33:09 490607140
>>490580
Ну твоё яскозал мало кого волнует, понимаешь ли
Аноним 31/12/19 Втр 13:11:00 490632141
Вот я задал вопрос >>490031 а аноны не ответили. Повторю - вокруг пламени возникает сфера толщиной в 0,4-0,7см где видно переход от фиолетового к красному. Ближе к пламени видно пропорц. к форме сферы белое свечение. Это оптические свойства воздуха, но каков именно процесс на уровне молекул/атомов?
Аноним 31/12/19 Втр 15:21:23 490636142
>>489328
я не понял пожалуста можно подробнее про атомный спектор? какие говне о чем ты
Аноним 01/01/20 Срд 01:56:12 490665143
Когда уже научатся точно определять уровень боли?
Аноним 01/01/20 Срд 05:30:44 490671144
>>490665
Чтобы понять насколько тупой пизде больно во время родов? Нахуй не нужно.
Аноним 01/01/20 Срд 13:26:46 490708145
Почему если (как еблан) поставить блюдечко с металлическим узором в микроволновку, то оно зашкворчит и нагреется, а в следующие разы ничего не происходит? Внешне блюдечко не меняется.
Аноним 01/01/20 Срд 13:56:30 490710146
>>490708
Там трещены в полосках металла появляются и контакт пропадает.
Они из длинной цельной полоски металла превращаются во много отдельных маленьких и перестают работать как антена.
Аноним 01/01/20 Срд 14:08:26 490711147
Аноним 01/01/20 Срд 14:25:43 490712148
До какой скорости может разогнаться корабль с таким вот термоядерным ракетным двигателем? Корабль 1000 тонн, топлива 900 тонн, удельный импульс двигателя 4 000 000 м/сек. Топливо нужно потратить столько, чтобы потом хватило для торможения до скорости 8 000 м/сек.
Аноним 01/01/20 Срд 14:27:26 490713149
>>490712
Не твой калькулятор.
Аноним 01/01/20 Срд 14:37:21 490714150
Неужели нет более или менее надёжного способа обезопасить водород в дирижабле? Ну, или юзать их чисто как грузовых роботов с минимумом людей на площадке? Просто дирижабли на гелии, по-моему, бесперспективны из-за его дороговизны и большей плотности. Водород из метана стоит копейки.
Аноним 01/01/20 Срд 14:41:28 490715151
Аноним 01/01/20 Срд 15:14:17 490716152
>>490714
Узнай концентрацию НЕ С КИСЛОРОДОМ И НЕ С ВОЗДУХОМ при которой водород хорошо восполняется, а при которой не очень, найди дешёвый инертный газ, найди насколько увеличится плотность и упадёт экономическая эффективность подъёмной силы.
Аноним 01/01/20 Срд 16:57:26 490737153
image.png (406Кб, 600x340)
600x340
Аноним 01/01/20 Срд 17:27:53 490766154
с новым годом .mp4 (2867Кб, 480x600, 00:00:29)
480x600
Аноним 01/01/20 Срд 18:07:19 490791155
image.png (1565Кб, 1920x1080)
1920x1080
image.png (1006Кб, 1000x750)
1000x750
>>490766
ЭКСТРЕННЫЕ ХРЕНОВОСТИ!
Недалеко от мухосранской губернии, нашли обезображенный труп местного Деда Мороза, двух оленей, и полуживую - Снегурку.
Пострадавшие, в своей сказочной колеснице, плыли по небу из пункта А в пункт B,
с целью доставить подарки детям, забрасывая их - в дымоходы.
14 пулевых ранений на теле пожилого человека, вызваны прямым попаданием из АК-47,
95% ожогов у Снегурочки- результат попадания в пиротехнические изделия и салюты на санях.
4 оленя - зажарены до смерти.

По словам местных жителей, накануне Нового Года, какие-то алкаши,
ВНЕЗАПНО, открыли окно, проорали пропитым голосом - "с новым годом",
после чего открыли беспорядочную очередь вблизи Луны, где пролетала эта парочка ничего не подозревающих жертв.
Миллиарды детей остались без подарков. Сейчас идёт следствие.
За убийство редкого представителя фауны, включённого в красную книгу - деда мороза, неспособного к регенерации,
злоумышленникам грозит штраф, в размере от надцати, до хуйста лярдов
необлагаемых налогами минимумов бесплатных подарков.
Снегурочка будет реабилитирована, но эта снегурочка уже не дурочка.
Аноним 01/01/20 Срд 18:47:54 490809156
>>490791
После нескольких мощных разрядов электрошокером-дефибриллятором, дед мороз оказался жив.
Сейчас он под аппаратами искусственного жизнеобеспечения, так как потерял много кока-колы.
Переливание кока-колы должно очухаться помочь старичку.
Автомат уже забрали, неадекватов аккуратненько, его очередью - положили.
Можете не ссать, и не рыпаться.
Аноним 01/01/20 Срд 22:35:22 490831157
Аноним 02/01/20 Чтв 04:19:38 490868158
Аноним 02/01/20 Чтв 17:30:25 490985159
Аноним 02/01/20 Чтв 17:41:45 490986160
>>490632
Короч. Молекулы могут быть в основном и возбужденном состоянии. Основное - это их обычное состояние. Возбужденное - это когда молекуле дали избыток энергии - нагрели. Чем сильнее нагреваешь, тем большую энергию молекула излучает обратно. Слабо нагрели - испускает ИК свет. Сильнее нагрели - светит красным. Нагрели очень сильно - светит синим. Загугли заебал. Химия-просто
Аноним 02/01/20 Чтв 17:49:27 490988161
>>489845
монитор - дифракционная решетка. Погугли дифракцию
Аноним 03/01/20 Птн 08:47:25 491062162
Почему НИИ и университеты, которые каждый год хуярят тонны макулатуры в виде статей, диссертаций по оптимизации технологических и бизнес-процессов, не могут внедрить хотя бы часть этой писанины у себя? Причем среди кучи мусора есть реально крутые вещи, но они вынуждены быть выброшены на помойку, потому что нахуй никому не нужны.
А на практике все дела ведутся на бумаге, информация нестуктурирована, электронная отчетность дублируется чисто ради галочки, чтобы не ебали сверху. Это как в каменном веке делать наскальные чертежи ракетных двигателей, которые, сцуко, правильные и будут работать, если их собрать на заводе. Но по факту все живут в каменном веке. Я не ебу, как такое возможно, и почему ничего не скатилось окончательно.
Аноним 03/01/20 Птн 10:19:54 491067163
У меня одноного такая херня с ютубом - если задать поиск по "Климатология" и включить фильтр "длиннее 20 минут", то в результатах поиска не будет НИ ОДНОГО русскоязычного видео?
Аноним 03/01/20 Птн 10:27:00 491069164
>>491067
Лол, я нашел всего одно видео на ютубе о климатологии на русском языке длиннее 20 минут, какого-то старого фрика увлекающегося народной медициной, иван чаем и снимающего очень важные видосики обо всем

https://www.youtube.com/watch?v=Aa7mF8ndBYA

Неужели в России не занимаются климатологией и не интересуются ей?
Аноним 03/01/20 Птн 10:44:17 491072165
Аноним 03/01/20 Птн 11:08:52 491077166
>>491072
Насколько же примитивное представление о климатологии у восточных славян, если ничего кроме Греты они о климатологии не знают.
Аноним 03/01/20 Птн 12:21:13 491085167
>>491077
Очень много знают. Ты просто академические знания ищещь в ютубе, где только Грета обитает.
Аноним 03/01/20 Птн 12:27:11 491087168
>>491085
Двачую. Залез на развлекательный портал в поисках истины
Аноним 03/01/20 Птн 12:29:44 491089169
>>491085
>Ты просто академические знания ищещь в ютубе, где только Грета обитает.
В Российском обитает один только треш, это так, да.
Аноним 03/01/20 Птн 18:27:20 491125170
>>491072
Анон, а есть где-нить прон с Гретой?
Аноним 03/01/20 Птн 19:09:16 491138171
>>491125
Или не прон, а терки половыми губками по члену, без проникновкния, вибратор через трусики, или просто эротика, да даже candy doll.
А то она угрюмая, пиздец, на гуглящихся пикчах. Не фапабельно, бля.
Аноним 03/01/20 Птн 19:46:58 491143172
image.png (3325Кб, 1200x1580)
1200x1580
image.png (579Кб, 480x480)
480x480
>>491138
подожди когда подрастет и раздвинет рогатку
Аноним 03/01/20 Птн 20:31:57 491152173
>>491143
Да иди ты, лучше уж пойду - в деревнях десятилетних поебу,
если не найду пятилетних с бешенством матки,
которые сами запрыгивают на хуец и просят войти в них - поглубже.
Аноним 03/01/20 Птн 20:45:32 491153174
.jpeg (5Кб, 225x225)
225x225
Мне, для некоторого произведения, надо изобразить знаменитых ученых в виде прстгспди "мемов"

Например Тесла и Эдисон. Их можно как Пепе и Филгая. Якобы Пепе-Тесла, потешается над Филгаем-Эдисоном. Так как последний проебал войну токов.

Надо еще про парочку придумать. Например бумера и зумера. Например какой-то ученый прошлого поколения, вывел там теорию какую-то. Какой-то зумерок, пытался ее опровергнуть но соснул. У Эйнштейна наверняка было много желающих наебнуть теорию относительности.

Или что-то другое. Но желательно известное в широких кругах событие. Которое можно "мемасно" обыграть.

Не обязательно даже конфликт. Просто как можно представить известных ученых.
Аноним 04/01/20 Суб 01:18:12 491165175
1566836130432.png (844Кб, 598x578)
598x578
Аноним 04/01/20 Суб 02:35:19 491167176
I.webm (1694Кб, 638x480, 00:00:16)
638x480
Правда ли, что сыворотка суперсолдата сделает реципиентов еще и умными, от того, что мозг перестанет разрушаться вследствие внешних факторов? Т. е. правда ли, что 90% забывания происходит потому, что солнечная радиация, усталость и вот это всё?
Аноним 04/01/20 Суб 08:02:42 491186177
шумер.png (217Кб, 252x454)
252x454
пепе.jpg (17Кб, 500x500)
500x500
>>491153
Возьми картинку с шумером, дальше ставь бумера, затем зумера. Типа эволюция)))))
Вот еще редкая геометрическая пепа, не благодари
Аноним 04/01/20 Суб 08:23:42 491192178
pie-chart-pepe-[...].jpg (27Кб, 800x800)
800x800
>>491186
> редкая геометрическая пепа
Ха, разве это редкая? Вот редкая.
Аноним 04/01/20 Суб 11:43:38 491199179
15781161281470.jpg (84Кб, 1126x644)
1126x644
Аноним 04/01/20 Суб 12:00:28 491200180
Во-первых, если тебе лекций, то иди нахуй:
https://youtu.be/MPr44oEJeCk
https://youtu.be/akv5f6W3LPg
https://youtu.be/fyVrDA4bLgE
https://youtu.be/X7lCCgahGkc
Во-вторых, если тебе фолк-климатологии про "ой, пиздец, пиздец", то иди в пизду.
В-третьих, отечественным климатологам не приплачивают за консенсус и не так интересно слушать, например, вклад какой обратной связи в альбедо от потепления победит -отрицательной от облаков и потепление замре на балансном уровне, или положительной от уменьшения площади поверхности снегов. На западе все обо все договорились, и можно пиздеть за красивую постапокалиптику в прямом эфире 24/7. А так, у нас много серьезной климатологии, особенно океанической. Ищи да обрящеш.
>>491153
Про Теслу какой-нибудь грустномем "когда выиграл войну токов, но остался нищим".
Аноним 04/01/20 Суб 12:01:40 491201181
Аноним 04/01/20 Суб 13:40:38 491202182
Газ может быть тяжелее железа? Он ведь по идее менее плотный.
Аноним 04/01/20 Суб 14:32:17 491203183
>>491202
Железо тоже может быть газом.
Абсолютно идиотски поставленный вопрос. Если мы об условиях комнатной температуры и давлении одна атмосфера, то, да, любая форма железа будет плотнее любого газа. Вообще, любой кристалл будет плотнее любого газа при прочих равных. Метаматериалы типа аэрогелей не рассматриваем.
Аноним 04/01/20 Суб 16:21:54 491207184
>>491202
На Юпитере газ под пиздецовым давлением, твердый как утюг
Аноним 04/01/20 Суб 17:02:39 491212185
>>491207
Это уже не газ. Алсо, пиздецовое давление можно превратить вещество не твердое тело, а в сверхтекучую жидкость, при определенных условиях.
Аноним 04/01/20 Суб 17:33:53 491216186
>>491212
Что-нибудь про сверхкритическое состояние слышал? Это газ.
>>491202
Условия... всё зависит от условий.
Аноним 04/01/20 Суб 18:02:56 491220187
>>491186
>>491200
Не, мне без надписей надо. Мне надо что-то вроде картинной галереи с учеными изобразить. Четыре портрета надо, но и не простых а кто там с физикой, электричеством и изобретательством связан и кто на слуху. Тесла и Эдисон уже есть. Надо бы еще парочку.
05/01/20 Вск 10:49:34 491383188
image.png (177Кб, 420x280)
420x280
image.png (275Кб, 800x400)
800x400
>>489321 (OP)
Это в обьективе дело? какой диаметр должен быть?
Аноним 05/01/20 Вск 10:53:19 491385189
>>491220
>Тесла и Эдисон уже есть. Надо бы еще парочку.
Максвелл и Фейнман.
Аноним 05/01/20 Вск 13:07:41 491409190
image.png (346Кб, 780x703)
780x703
На двачах нашел картинку. Она имеет какое-то отношение к реальности?
Аноним 05/01/20 Вск 13:09:32 491410191
>>491220
Кидай телегу или вк, обкашляем, запилю на фш за символическую плату
Аноним 05/01/20 Вск 13:16:01 491412192
>>491409
>На двачах нашел картинку. Она имеет какое-то отношение к реальности?
Кому и двач - реальность.
Есть ВОЗ и есть ее доклады. Хочешь - читай, не хочешь - жди умный ответ на тупой вопрос.
HIV/AIDS surveillance in Europe 2019 https://apps.who.int/iris/bitstream/handle/10665/329997/9789289054607-eng-rus.pdf
Аноним 05/01/20 Вск 13:43:09 491419193
Аноним 05/01/20 Вск 18:03:26 491445194
Сап, наукач, есть парочка вопросов:
1) Из имеющихся технологий клонирования (хотя правильнее сказать близнецирования) уже есть такие, которые позволяют использовать клетку кожи или волоса для создания клона? Насколько правильно я это понял из статьи про китайских обезьян Чжун и Хуа?
2) Насколько сложно при этом процессе изменить пол клона на противоположный донору?
Карамельное ракетное топливо Аноним 05/01/20 Вск 19:13:03 491456195
Аноним 05/01/20 Вск 20:34:36 491461196
>>491456
Ты уверен что 6 к 1? Что у тебя в продуктах?
Аноним 05/01/20 Вск 22:55:41 491469197
image.png (31Кб, 606x223)
606x223
>>491461
даже 7:1 получилось
может, пора химию повторять просто?
Аноним 06/01/20 Пнд 03:56:07 491493198
>>489321 (OP)
Может ли быть гравитация - многомерной, работать и описываться - в более чем 4-х измерениях?

На эту мысль наталкивает тот факт, что гравитация, каким-то образом,
могла сдерживать всё расширяющееся четырёхмерное пространство-время
в космологической сингулярности, то есть - в планковском объёме!
А какое там, внутри, может быть измерение? Там вообще, походу, нет никаких измерений.
Значит, возможно, с какого-то дополнительного измерения всё это сдерживалось гравитацией,
пока она не отделилась, в планковскую эпоху...

Быть может выход за пределы четырехмерного пространства-времени,
позволит создать квантовую терию гравитации, чтобы описать гравитацию в большем числе измерений?
Аноним 06/01/20 Пнд 04:12:44 491494199
f1264[1].JPG (23Кб, 435x387)
435x387
>>491493
Если существует пятое измерение,
то гравитационное искривление пространства-времени
в чёрных дырах и космологической сингулярности,
может быть вовсе не искривлением, а выгибанием его в пятимерное пространство, как выгибание центра араболоида на пикрил.
Тогда, проекция этого выгнутого континуума на чётырехмерное пространство,
будет конформно уменьшаться, в четырёхмерии порождая иллюзию искривления континуума.

Существование пятого пространственного измерения очень трудно представить себе,
интуитивно, из-за перпендикулярности прямых углов...
А что, если пятое измерение, какое-нибудь - временное?
Может ли гравитация как-то менять параметры такого измерения?
Аноним 06/01/20 Пнд 04:30:51 491495200
>>491494
Ты чего не спишь? Прими свои таблетки и спать ложись.
Аноним 06/01/20 Пнд 06:18:20 491501201
>>491495
У меня чё-т от Пенроуза - петлевая квантовая гравитация теперь. Так что петляй отсюда, быренько.
06/01/20 Пнд 19:54:57 491550202
image.png (1080Кб, 1280x622)
1280x622
image.png (484Кб, 1200x519)
1200x519
>>491383
в увеличительной линзе
Пикрил 1 - фото без увеличения (как видит человеческий глаз)
Пикрил 2 - фото с увеличением и обрезкой

Вот тебе и десятикратное изменение соотношения размеров объекта
06/01/20 Пнд 20:02:44 491551203
>>491062
патенты работают, например, это не бумажки.
новые технологии разрабатываются - но изготовить технологию - это один порядок средств, сделать прототип - другой порядок финансирования, организовать потоковое производство - третий порядок. ИЧСХ ты пользуешься OLED дисплеями, которым 20 лет от роду
лекарства - одно дело синтезировать вещество, убивающее рак в пробирке, другое дело - убить на тестах сотни кроликов и десятки людей. это дорого. но опять же, если идея годная - то у нас появляются новые лекарства
Аноним 06/01/20 Пнд 23:42:22 491562204
>>491494
Долбаёбик, это всё математические абстракции чтобы было легче решать/математические абстракции при которых расчёты с наблюдениями сходятся.
Аноним 06/01/20 Пнд 23:43:27 491563205
>>491493
И нет никаких "много измерений".
Аноним 06/01/20 Пнд 23:46:15 491565206
>>491383
По-твоему первая картинка это фотка а не картинка?
Аноним 07/01/20 Втр 02:52:41 491580207
>>491563
>И нет никаких "много измерений".
Не факт.
Аноним 07/01/20 Втр 04:09:45 491582208
Аноним 07/01/20 Втр 04:24:31 491583209
Аноним 07/01/20 Втр 05:11:05 491586210
>>491583
Гуги планковское время, планковскую длину и планковский объём.
Меньше их - нихуя нельзя измерить, а значить нет оснований полагать даже, что меньше их что-то есть.
Аноним 07/01/20 Втр 06:40:38 491596211
>>491586
>Гуги планковское время, планковскую длину
И с чего ты решил, что эта математическая идея доказывает, что пространство и время дискретно?
Аноним 07/01/20 Втр 10:45:12 491608212
Правда что механизмы маркетинга и рекламы в интернете давно уже работат в том числе за счёт взаимодейтвия с дофаминовым циклом. Т. е. реклама и лайки в соц. сетях как наркотик действуют и создают положительную обратную связь на себя, вынуждая всё больше делать что-либо(ставить лайки, покупать что-нибудь).
Аноним 07/01/20 Втр 13:19:02 491619213
>>491608
Тупые говноеды всегда жрут говно.
Аноним 07/01/20 Втр 13:20:10 491620214
>>491586
Даунич, ты совсем не разбираешься в теме и что-то пиздишь.
Ещё и гуглить советуешь, лол
Ты сам хоть пробовал гуглить планковские величины?
Аноним 07/01/20 Втр 13:52:26 491628215
>>491608
>Правда что
Если у тебя есть разум и критическое мышление, то никакой маркетинг на тебе не сработает.
Аноним 07/01/20 Втр 14:33:09 491631216
Аноним 07/01/20 Втр 15:55:02 491632217
156574514645609[...].jpg (352Кб, 700x1031)
700x1031
Сколько примерно энергии необходимо андроиду типа терминатора, если он целый день бегает и пиздит морды? Это эквивалент скольки телефонных аккумуляторов в день? Как по мне это вообще почти нихуя и какой-то ядерный принципиально новый источник питания ни к чему. Автомобильного аккума хватит на несколько месяцев, а его и подзарядить можно.
Аноним 07/01/20 Втр 18:36:31 491636218
>>491632
Смартфон, который не преодолевает силу гравитации и не обрабатывает видео высокой четкости без перерыва, разряжается за день. Какой андроид, мань?
Если в цифрах: обычному человеку, даже если он не бегает и не пиздит морды целый день, нужно около 2000 калорий в сутки. Это около 8000 Вт*ч, иначе говоря, примерно 3 телефонных аккумулятора или 1/10 твоего автомобильного аккумулятора, которого
>андроиду типа терминатора
и на неделю не хватит.
Аноним 07/01/20 Втр 20:23:35 491644219
>>491469
Советую написать нормально реакцию. Нитрит у него.... Пиздец.
дядя пиши до азота, калий в карбонат пиши
Аноним 07/01/20 Втр 20:26:11 491645220
>>491493
Советую изучить понятие "мат. модель"
Аноним 07/01/20 Втр 22:25:16 491651221
image.png (19Кб, 633x156)
633x156
>>491644
о, даже не подумал о распаде нитрита. теперь правильно?
и все еще далеко до 13/7
Аноним 07/01/20 Втр 22:52:25 491654222
Сап, двачеры. Такой вопрос: Если гравитация влияет на скорость течения времени, значит где-то во вселенной есть место с идеальной скоростью течения времени?
Аноним 07/01/20 Втр 23:27:30 491657223
>>491654
Гравитация влияет на скорость физических процессов. Время только у тебя в голове. Но если рядом нет сверхмассивных объектов, то время будет идти примерно как в остальной наблюдаемой вселенной, где нет высоких масс, да.
Аноним 07/01/20 Втр 23:33:30 491659224
>>491657
Но ведь вся Вселенная, в совокупности своей - пиздато массивна, и гравитационные волны накладываясь друг на друга интерферируют гравитацией гравитирующей.
Аноним 07/01/20 Втр 23:34:59 491660225
>>491659
Скорее гравитируют гравитацией гравитациоаналистичной.
Аноним 07/01/20 Втр 23:36:04 491661226
>>491660
Если чётко по терминологии, то гравитонациоаналичной.
Аноним 07/01/20 Втр 23:57:46 491666227
>>491659
Да и похуй, мы примерно одинаково подвергнуты гравитации как и другое место не рядом с черной дырой.
Аноним 08/01/20 Срд 02:00:40 491680228
>>491636
> нужно около 2000 калорий в сутки
а учитывая, что мощность, работа и сила связаны прямолинейно, то нужно умножить на: различие в весе. Терминатор весит 150 кило. То есть 4000 ккал. Учитывая подвижный образ жизни: 7000 ккал. Учитывая работу 24\7 накидываем ещё 30%. Ну примерно 10 000 ккал в день. 40 000 Вт*ч. 0.5 аккумулятора в день.
А если он ещё и из лазера будет стрелять, то, учитывая опыт factorio, его хватит на 2 выстрела
Аноним 08/01/20 Срд 03:36:22 491685229
>>491680
Терминатор весит 250 кило, T-X — ~200

Эксперт по терминаторам

А ядерные топливные элементы на 200 лет работы (1 шт. — 100 лет) не звучат как сказка?
Аноним 08/01/20 Срд 03:39:53 491686230
Ах да, 98% энергии человеком тратится на обеспечение органов, ученые комнатные. У робота только питание на сервоприводы и процессор, так что минус 98% по человеку для расчетов на робота.
Аноним 08/01/20 Срд 06:05:13 491694231
>>491651
Да. Теперь правильно. 13/7 это ~1,86, понятно, что реакция идёт хуй пойми как с кучей лишних продуктов, поэтому окислителя берут побольше, чтобы наверняка.
Аноним 08/01/20 Срд 06:07:05 491695232
>>491680
https://habr.com/ru/post/98489/
>XeF8 <-> XeF2
> вся гигантская энергия от давления в 70 ГПа переходит в химическую энергию молекулярных связей.
>Профессор химии Вашингтонского университета Чонг-Шик Ву (Choong-Shik Yoo) говорит,
>что это самая концентрированная форма хранения энергии после ядерных материалов.

Откуда нам знать, что таким вот образом, сдавливая вещества, нельзя получить какие-то более плотные формы каких-нибудь кристаллов,
или даже квазикристаллов, которые смогут хранить в себе энергию давления сколь-угодно долго,
а высвобождая энергию при расжатии (например при нагревании), выдавать тугую струю экологически-безопасных веществ,
например струя кремния, струя песка, струя воды, струя кислорода?
Если такая шняга будет создана, то прицепив турбину можно будет вообще - портативную электростанцию в себе носить. Лол.
Аноним 08/01/20 Срд 06:49:10 491696233
>>491695
>Откуда нам знать, что таким вот образом, сдавливая вещества, нельзя получить какие-то более плотные формы каких-нибудь кристаллов,
>или даже квазикристаллов, которые смогут хранить в себе энергию давления сколь-угодно долго

Ты не поверишь. Но такие вещества уже давно были придуманы. Знаешь как называются? Взрывчатые.
Аноним 08/01/20 Срд 07:03:43 491697234
>>491695
>Откуда нам знать, что таким вот образом, сдавливая вещества, нельзя получить какие-то более плотные формы каких-нибудь кристаллов,

https://www.youtube.com/watch?v=R0zwwbcWcNY
Аноним 08/01/20 Срд 07:07:34 491698235
>>491680
>10 000 ккал в день. 40 000 Вт*ч. 0.5 аккумулятора в день

Физика уровня /b/
Аноним 08/01/20 Срд 09:03:34 491703236
>>491696
Эти взрывчато ебошат, резко, быстро, ну пиздец...
Надо чтобы плавно было, чтобы как струя, как ссыт конец!
Лишь чуток прогрел хернюшку - никакого, блядь хлопка,
а струя попёрла туго, на турбину у меня.
Генератор закрутился, ток пошёл по проводам, в экологии всё чисто, безопасно дохуя.
А юзать фторид ксенона - нет годнейшего резона,
фтор не только ядовит, он ещё в воде горит.
Аноним 08/01/20 Срд 09:13:05 491704237
>>491703
>фтор не только ядовит, он ещё в воде горит.
Фтор нам больше ни к чему
Полетим мы на Луну
Гелия там наберем
И в Кашиик все рванем
Аноним 08/01/20 Срд 09:32:22 491707238
>>491704
Гелий-3 надо копать, реголит же ёпта, блядь...
Там его почти что нет, не нарваться б на минет.
Гелий проще нам достать, на Юпитер бы слетать!
На Сатурне ещё есть, там же газовая смесь...
Стоит лишь отфильтровать, гелий-3, едрёна мать.
Если в синтез сможем там, заебись же будет нам,
атмосферу фильтровать, и в реакторе сжигать.
Только легким должен быть наш реактор, чтоб парить...
Но добыть бы гелий-3, с водорода, из воды!
Никуда чтоб не летать, радиация же, блядь...
Аноним 08/01/20 Срд 10:05:37 491710239
>>491707
До Луны рукой подать, в Сатурн ебена мать!
До него лететь устанем, будем кишки высирать
Так что гоу на Луну и устроим кутерьму
Аноним 08/01/20 Срд 10:38:01 491713240
image.png (178Кб, 623x349)
623x349
>>491710
Что там делать на Луне? Без магнитов - как в говне.
Там нет поля от частиц, прущих с Солнца на тупиц.
Ветром сдует только так, вызывая спидорак,
радиация там прёт - гены с ДНК сотрёт.
Реголит копать решил? Чем Сатурн не угодил?
Гравицапу заведи, Алькубьерре подключи,
парус солнечный открой, только мощный, развитой,
ускоренье обрети, роботов там посади,
как накопят гелий-3, выстел бочкой до Земли,
сразу мутишь перехват, точно расчитав квадрат,
парашют потом открой, чрез сигнал беспроводной,
бочку оземь посади, и в реакторе год жги.
Аноним 08/01/20 Срд 14:11:25 491734241
image.png (22Кб, 773x161)
773x161
>>491694
ладно, я нашел
там еще сложнее, но с соотношением теперь все понятно
Аноним 08/01/20 Срд 14:23:23 491737242
>>489321 (OP)
Электромагнитного поля нет, есть магнитное поле и электроны в электронном поле на которые оно(магнитное) воздействует?
Аноним 08/01/20 Срд 15:52:57 491739243
>>491737
>Электромагнитного поля нет
Вроде как есть и квант его фотон.
>есть магнитное поле
Магнитное поле это составляющая электромагнитного поля.
>электроны в электронном поле на которые оно(магнитное) воздействует?
Ага.
Аноним 08/01/20 Срд 15:56:01 491740244
>>491739
Какая (нах) электрическая составляющая в электромагнитном поле кроме магнитной, и нах она вообще нужна, если она себя никак не проявляет?
Или проявляет?
Аноним 08/01/20 Срд 15:56:07 491741245
>>489321 (OP)
Поясните за Асю Казанцеву. В ее выступления серьезных ошибок нет (касательно основных предметов обсуждения)?
https://youtu.be/sI5d5lBwn8c
мне понравилось это (несмотря на неприятную картавость), хочется и другими ее лекциями обмазаться
Аноним 08/01/20 Срд 16:24:24 491742246
>>491740
>>491737
Электромагнитное поле не разделимо, это единая сущность. Иногда оно проявляет себя как то, что исторически называется эффектами электрического поля, иногда - как эффекты магнитного поля, чаще всего - и тем, и тем. Это легко видеть даже с уровнем образования первокурсника, если посмотреть, как меняются электрическое и магнитное поля при смене системы отсчёта.

>электроны в электронном поле
Электронное поле и электромагнитное поле это разные понятия (фундаментально разные, потому что одно - фермионное, а другое - бозонное). Электронное поле - это то, как описываются электроны в квантовой теории поля. Электромагнитное поле - это то, как описываются фотоны в квантовой теории поля (а в классической и квазиклассической физике - старое-доброе "школьное" поле с Кулоном, Лоренцом, Фарадеем, и Максвеллом).
Аноним 08/01/20 Срд 16:33:01 491744247
>>491742
Пустое бла-бла-бла, ты ни на что не ответил, никакой информации не написал. Пустая вода, фу.

Будто высер гуманитария прочитал, фу-фу-фу.
Аноним 08/01/20 Срд 16:55:56 491745248
>>491744
Это тред тупых вопросов.
По постам задающего (тебя?) мне было видно, что анон не шарит ну вообще ни в чём. Смысл его (тебя?) грузить?
Ты, собственно, эталонный щитпостер, который засрал весь /sci/, из-за чего фактически стоит отвечать и помогать только в /math/.

Вот тебе строгий ответ - необходимость существования обоих полей следует из Лоренц-инвариантов тензора Фарадея - они показывают, что нельзя поменять СО так, чтобы не-нулевое электрическое поле исчезло. Необходимость существования электромагнитного поля в сущности же следует из требования локальной U(1)-инвариантности уравнения Шрёдингера, потому что это с необходимостью предполагает существование векторного потенциала, который играет роль связности в ковариантной производной.

Естественно, ты ни-ху-я-шень-ки из такого объяснения не поймёшь, поэтому я тебе и ответил словами, которые ты ещё может быть осилишь. А именно, поменяй СО и посмотри на компоненты.

Так что не знаешь - сиди и помалкивай.
Аноним 08/01/20 Срд 17:03:16 491746249
>>491745
То что тред тупых вопросов, и если человек не шарит это не повод НЕ ДАВАТЬ ему ответ вообще, высирая кучу пустого текста.
Аноним 08/01/20 Срд 17:04:46 491747250
>>491745
Блять, какой же ты убогий чмошник, просто тошно.
Что в том посте-гуманитарном пустом высере была жалкая попытка выебнуться, что в этом.
Какой же ты жалкий.
Аноним 08/01/20 Срд 17:05:07 491748251
>>491746
>высирая кучу пустого текста.
Я дал конкретный совет, что делать ручками, а именно
>Это легко видеть даже с уровнем образования первокурсника, если посмотреть, как меняются электрическое и магнитное поля при смене системы отсчёта.
Так что ты читаешь жопой. Пиздуй в дотку погоняй, в /b/ рулеточку покрути.
Смотрит в пост - нихуя не видит, а другие виноваты, охуеть теперь.
Аноним 08/01/20 Срд 17:05:46 491749252
Аноним 08/01/20 Срд 17:06:23 491750253
>>491745
Ты настолько тупой, что даже не можешь понять вопроса написанного простыми словами.
Было спрошено КАК ОНА ПРОЯВЛЯЕТСЯ, а ты притащил лепет о математических абстракциях и фантазиях.
Аноним 08/01/20 Срд 17:09:23 491751254
>>491749
>>491748
Ахах, у убого чмошника не получилось выебнуться, он порвался и разнылся.
Аноним 08/01/20 Срд 17:47:38 491752255
>>491737
>Электромагнитного поля нет
Электромагнитное поле есть. Выражается оно ненулевым тензором, часть компонент которого иногда называют электрическим полем, а часть магнитным. Раздельно они не существуют.
Аноним 08/01/20 Срд 19:24:57 491755256
>>491752
Это всё абстракции, мотематика, циферки.
Где проявляется электричкая составляющая?
Магнитная проявляется, магниты магнитят, магнитное поле заставляет двигаться электроны, при бОльшем напряжении-большем магнитном поле, через проводник/нагрузку может пройти больше электронов и совершить больше работы. И т.д.
А электрическая составляющая как и где проявляется?
Аноним 08/01/20 Срд 19:40:14 491760257
>>491755
Хмм, а вот кстати, электроны в проводниках пихают друг-друга?
Т.к. обладают одинаковым зарядом?
Аноним 08/01/20 Срд 20:03:07 491763258
>>491755
>Магнитная проявляется, магниты магнитят, магнитное поле заставляет двигаться электроны, при бОльшем напряжении-большем магнитном поле, через проводник/нагрузку может пройти больше электронов и совершить больше работы. И т.д.
Перехожу в движущуюся систему отсчета и получаю:
магниты электрисят, элетрическое поле заставляет двигаться электроны, при бОльшем напряжении-большем электрического поля, через проводник/нагрузку может пройти больше электронов и совершить больше работы.
Аноним 08/01/20 Срд 20:10:17 491764259
>>491755
Вообще вопрос максимально странные(тупой), так как любом школьнику знакома формула для силы действующей на частицу в магнитном или электрических полях. Соответственно, спрашивается: хули тут непонятного то?
Аноним 08/01/20 Срд 20:48:35 491766260
>>491755
>Где проявляется электричкая составляющая?
Заряды тянутся друг к другу. Прикинь
Аноним 08/01/20 Срд 21:06:27 491769261
>>491763
Не, твой мозг совсем поплыл от промытия херовенькими абстракциями.
Аноним 08/01/20 Срд 21:07:55 491770262
>>491764
Максимально тупой ответ, учитывая что вопрос был не про школьные формулки.
Соответственно, спрашивается: хули в вопросе непонятного то?
Аноним 08/01/20 Срд 21:09:19 491771263
>>491766
О, хорошо, неплохо.

Хотя не уверен что совсем то...

Аноним 08/01/20 Срд 21:12:43 491773264
>>491760
А это чё проигнорили? Это так?
Аноним 08/01/20 Срд 21:14:16 491774265
>>491773
Что означает "пихают"? Они же не шарики тебе, они там в виде ферми-газа в кристаллической решетке.
Аноним 08/01/20 Срд 21:15:40 491775266
>>491774
Шарики, шарики, размытые, с неясным размером, но взаимодействующие между собой.

Да и газ это тоже шарики, и тоже пихающие себя.
Аноним 08/01/20 Срд 21:17:52 491776267
>>491775
Если под "пихают" ты имеешь ввиду взаимодействие, то конечно.
Аноним 08/01/20 Срд 21:20:47 491777268
>>491776
Получается, может быть такое предельное напряжение, при котором проводник разорвёт?
И такое, потом, которое начнёт вырывать электроны уже из атомов и разрушать структуру вещества.
Аноним 08/01/20 Срд 21:22:31 491778269
>>491777
Электроны вырывать из него проще конечно (прочитай про катод-анодную трубку). Чтобы разорвать, надо дохуя приложить.
Аноним 08/01/20 Срд 21:24:09 491779270
>>491778
Я когда тут раньше задавал такой вопрос, когда не додумался опустится на более низкие уровни абстракции/микромира.
Так надо мной ржали.
Пидоры.
Аноним 08/01/20 Срд 21:27:38 491780271
>>491778
Но если же не будет особого тока то они должны быстро обратно на место ловить другие вместо вылетевших, и разорвать в принципе можно.

Хотя мало электронов это и малое распихивание по идее.
А больше тока, больше электронов, больше распихивание.
Наверное должно быть какое-то оптимальное соотношение ток-напряжение, для разрывания.
Аноним 08/01/20 Срд 21:29:36 491782272
>>491780
У тебя плавится начнет при высоком токе. Так как греется всё. Если хочешь устроить именно разрыв, то надо сверхпроводник разгонять.
Аноним 08/01/20 Срд 21:33:09 491783273
>>491782
Я знаю. Я и говорю про оптимальное соотношение.
И я хочу разрыв от напряжения, а не от тока.
Если много электронов то.. Хотел написать что это будет уже не то, но pvt...
Хз, надо подумать.
Аноним 08/01/20 Срд 21:33:58 491785274
>>491782
Применимо ли в этом случае отношение pvt?
Аноним 08/01/20 Срд 21:38:26 491787275
>>491782
И тут тоже кстати странно, знал что сверхпроводники могут разрываться от большого тока, и знал что в сверхпроводниках ток течёт вне их, снаружи.
Но знал я это как-то отдельно друг от друга, а вместе об этом только сейчас подумал.

Так как тогда так получается?
Аноним 08/01/20 Срд 21:49:49 491789276
>>491770
>Где проявляется электричкая составляющая?
>приводишь выражение где она явно входит
>тупой ответ
Ну я хуй знает.
>хули в вопросе непонятного то?
В том, что вопроса нет.
>>491769
Жалко только, что эти абстракции подтверждаются экспериментом.
Аноним 08/01/20 Срд 22:13:58 491796277
>>491789
>Жалко только, что эти абстракции подтверждаются экспериментом.
Какой же ты тупой.
Аноним 08/01/20 Срд 22:15:00 491797278
Аноним 08/01/20 Срд 22:23:25 491800279
>>491797
Ну не плачь, тупой даунёнок.
Аноним 09/01/20 Чтв 00:18:13 491813280
Через примерно 1,1 млрд лет от настоящего времени, наше дневное светило будет ярче на 11 %, чем сейчас. Это приведет к существенным климатическим изменениям на Земле и вымиранию большинству живых существ. Несмотря на это, жизнь может остаться в океанах и полярных областях. Интересно, что в этот момент, самой благоприятной для жизни планетой станет Марс


В каком году начнутся экспедиции. Как будет происходить терраформирование. Придется ли генетически изменять колонистов. Какие мысли есть на этот счет?
Аноним 09/01/20 Чтв 01:17:13 491819281
>>491744
Вообще-то он все очень четко и понятно ответил. Если ты не понял, то может проблема не в нем, а в тебе?
Аноним 09/01/20 Чтв 01:25:36 491820282
>>491745
>Естественно, ты ни-ху-я-шень-ки из такого объяснения не поймёшь,
Проблема школьного образования. По-моему в современных школах вскользь рассказывают о Фарадее, о том великой революции научной мысли которую он инициировал введя само понятие "поля", в школе не рассказывают о том что такое квант и зачем Макс Планк ввел это понятие в науку, не рассказывают о гамильтониане Дирака, очень вскользь касаются теории гравитации Эйнштейна и т.д.
Аноним 09/01/20 Чтв 01:26:41 491821283
>>491755
>Это всё абстракции, мотематика, циферки.
Вообще-то нет, ты сам в этом можешь убедиться.
Аноним 09/01/20 Чтв 01:47:17 491822284
Аноним 09/01/20 Чтв 01:50:20 491823285
>>491654
Ещё по теме спрошу, получается что люди на экваторе стареют быстрее или медленнее людей на полюсе из-за разницы в скорости вращения?
Аноним 09/01/20 Чтв 02:04:11 491824286
>>491823
Ускорение свободного падения 9,78 м/с2 на экваторе и 9,83 м/с2 на полюсах. Следовательно для наблюдателя на экваторе, часы на полюсах будут идти медленнее или быстрее, как думаешь?
Аноним 09/01/20 Чтв 02:28:59 491828287
Такой вопрос назрел. Структуру магнитов рентгеновскими лучами уже изучали?
Аноним 09/01/20 Чтв 03:31:52 491830288
>>491824
Медленнее, но за счёт СТО разницы не будет?
Аноним 09/01/20 Чтв 04:29:18 491834289
Магнитное поле имеет четкие границы, или просто убывает по удалению от источника? Минимальное значение МП считается границей МП?
Аноним 09/01/20 Чтв 05:38:28 491839290
Аноним 09/01/20 Чтв 06:34:30 491849291
>>491744
>>491747
>получил ответ
>"кококо вода гуманитарий!"
>получил строгий ответ
>"слишком сложна нипонятна, решил выебнуться прост!"
Хлебушкам в науку дорога заказана. Как ты сумел обосраться в треде для начинающих? Если что, это тред тупых вопросов, а не тупых постеров, так что шагай мимо.
Ещё и семёнит в ответах, вообще охуеть. Обос/sci/, который мы заслужили.
09/01/20 Чтв 19:33:43 492011292
>>491565
в сайте наса разместили. они фотошопят?
09/01/20 Чтв 19:40:18 492012293
йцуйцуйцу.png (2867Кб, 2490x1440)
2490x1440
Аноним 09/01/20 Чтв 20:37:40 492015294
15073233833050.jpg (374Кб, 900x1200)
900x1200
>>489321 (OP)
Почему железо в воде ржавеет быстрее? Оно же окисляется, а в воде кислорода меньше, значит должно наоборот.
НЕСТЫКОВОЧКА!
Аноним 09/01/20 Чтв 20:42:36 492016295
>>492015
вода очень хороший растворитель.
там не так просто все выглядит как ты думаешь, короче.
Аноним 09/01/20 Чтв 20:44:52 492017296
>>492016
>вода очень хороший растворитель.
и че? если она растворяет метал то просто метала должно становиться меньше но он не должен ржаветь
Аноним 09/01/20 Чтв 20:49:10 492018297
>>492015
Потому что если в воде есть растворенный кислород, а он есть, то железо будет окисляться. Если его не будет, то ничего быть не должно - по идее. Ну так вроде?
Аноним 09/01/20 Чтв 20:51:20 492020298
>>492018
но в воде кислорода меньше чем в воздухе, тогда почему в воде окисление идет быстрее?
Аноним 09/01/20 Чтв 20:55:35 492022299
>>492020
Окисление идет если есть и кислород, и вода. Тут меньше/больше роли не играет в абсолютном значении.
Аноним 09/01/20 Чтв 20:59:10 492023300
>>491828
>>491834
Аноны, пожалуйста, я жду. Особенно интересует первое, второе вроде понятно, но тоже хотел бы услышать ответ. Простите за лишний пост.
09/01/20 Чтв 21:51:18 492027301
>>492015
вода кислота неплохая.
алсо, почитал комменты, итт дебилы сидят, зачем сидят хз, я вопрос задал недавно. так что уйду уже, а эти чё делают здесь хз
Аноним 09/01/20 Чтв 22:20:32 492030302
>>492017
Если воздух сухой, то железо покрывается оксидом, а он плотный сука, и закрывает железо от воздуха. Если воздух влажный, то железо окисляется до оксогидроксида, а эта сука пористая, и дает доступ воздуху дальше окислять.
Аноним 10/01/20 Птн 01:03:51 492041303
>>491834
>Магнитное поле имеет четкие границы, или просто убывает по удалению от источника?

Поля заполняют все пространство, без исключения так что границ нет.
Аноним 10/01/20 Птн 01:04:12 492042304
>>491828
>Структуру магнитов рентгеновскими лучами уже изучали?
Конечно. Еще в прошлом веке.
Аноним 10/01/20 Птн 01:08:49 492043305
>>492041
Отлично ответил, нихуя не скажешь.
>>492042
Мне нужны результаты исследований. Не гуглится.
Аноним 10/01/20 Птн 01:15:59 492044306
>>492022
>Окисление идет если есть и кислород
Окисление химический процесс, сопровождающийся увеличением степени окисления атома окисляемого вещества посредством передачи электронов от атома восстановителя (донора электронов) к атому окислителя (акцептору электронов).
Аноним 10/01/20 Птн 01:17:39 492045307
Аноним 10/01/20 Птн 01:21:41 492047308
>>492044
Ты хочешь сказать что я должен был отвечать этим на тот вопрос?
Аноним 10/01/20 Птн 01:52:01 492049309
e
Аноним 10/01/20 Птн 08:13:00 492068310
>>492045
>он не знает разницу между интернет архивом и архивом научных статей
Хотел выебнуться, а вместо этого ты обосрался.
Аноним 10/01/20 Птн 08:44:48 492071311
>>492027
>вода
>кислота
5 баллов Кельвина этому гриффиндорцу от абсолютного нуля по знании химии.
Аноним 10/01/20 Птн 10:20:41 492077312
Аноним 10/01/20 Птн 12:09:03 492117313
Аноним 10/01/20 Птн 12:45:13 492130314
>>492071
>карбонатов не существует
>гидрокарбонатов нет в природе
>угольная кислота не существует при н.у.
Бог науки итт, поделись ещё своей модрустью)
Аноним 10/01/20 Птн 12:47:36 492131315
image.png (21Кб, 621x363)
621x363
>>492015
Просто процесс ржавления сложнее, чем обычная хим. реакция.
Аноним 10/01/20 Птн 12:51:38 492133316
>>491834
> просто убывает по удалению от источника?
>this
в степени квадрата или куба от расстояния убывает
вон, даже уравнение Шрёдингера говорит, что есть шанс найти электрон от атома в другом конце вселенной
>Минимальное значение МП считается границей МП?
Да. Для уравнения Шрёдингера эта граница - где электрон находится 90% времени. С магнитным полем, думаю, попроще. Его граница определяется объектами исследования и погрешностью, которая появится, если сказать "всё, дальше поле не влияет"
Аноним 10/01/20 Птн 12:53:24 492134317
Аноним 10/01/20 Птн 12:54:43 492135318
>>491823
> люди на экваторе стареют быстрее или медленнее людей на полюсе
старение - биохимический процесс. Как и любая химическая реакция, скорость зависит от температуры - чем выше, тем обычно быстрее. Соответственно, на севере старение идёт медленнее, что подтверждается статистикой о средней продолжительности жизни
Аноним 10/01/20 Птн 12:56:58 492137319
>>491813
> В каком году начнутся экспедиции. Как будет происходить терраформирование. Придется ли генетически изменять колонистов. Какие мысли есть на этот счет?
читай статьи. Мнений много, и все они на грани научной фантастики. Но проблемы Марса очевидны.
Радиация в полёте и на планете
Отсутствие атмосферы - её будет сдувать без магнитного поля
и как это решить - я хз
Аноним 10/01/20 Птн 12:57:05 492138320
Вопрос про мышцы человека. На ютубе есть много видео о том как накачаться. В некоторых говорят что сила зависит не только от размера волокна и количества белка в нём, но и от так называемой нервной проводимости. Сейчас её редко вспоминают, но в 2015 регулярно упомянали, даже ссылались на старые книги. Я так же знаю, что у людей с тяжёлыми заболеваниями, типа эпилепсии, во время припадков бывает меняется голос в т. ч. на "нечеловеческий", и это объясняется как раз тем, что человек потерявший контроль над собой сокращает мышцы горловых связок таким образом, каким это сделать невозможно для него в нормальном состоянии. Аналогично с людьми которых бьёт током: бывает что человек не может самостоятельно отпустить провод под напругой т.к. из-за тока по телу мышцы сильно сокращаются, и даже два человека с трудом могут его отдёрнуть. Т. е. выходит что у мышц есть состояния такого напряжения, в которые их нельзя привести обычным усилием воли. Вопрос можно ли как-то научится осознанно сокращать мышцы на столько же быстро и сильно ?
Аноним 10/01/20 Птн 13:00:16 492139321
>>491763
чтобы так было, система отсчёта должна быть разогнана до релятивистских скоростей. мне кажется, на этом этапе может что-то сломаться
Аноним 10/01/20 Птн 13:04:30 492141322
>>491760
> электроны в проводниках пихают друг-друга
если взять просто провод, то там 2 процесса:
1) притягивание - из-за того, что ядро заряжено положительно, а электроны отрицательно
2) отталкивание - из-за того, что ты сказал
И поэтому в проводе возникает равновесие - эти 2 процесса выравниваются и атомы находятся на строго определенном расстоянии друг от друга.

Когда же ты по проводу пропускаешь напряжение, то электроны по проводу не движутся)
Просто один электрон с одного атома прыгает на второй, выталкивая со второго атома электрон, который прыгает на третий атом, выбивая из него электрон, который прыгает дальше
Ещё в этом деле замешаны дырки, но я не знаю как. Мб аноны знают?
Аноним 10/01/20 Птн 13:11:14 492144323
>>492138
> у мышц есть состояния такого напряжения, в которые их нельзя привести обычным усилием воли
Тяжёлоатлеты регулярно рвут себе связки, потому что напрягают мышцы до самого предела при помощи обычного "усилия воли".

Всё, что ты сказал, имеет мало чего общего с реальностью, но если хочешь, вот штуки, которые помогут при одинаковых мышцах одному человеку уделать другого:
1) Повышенное кровяное давление - больше воздуха поступает к мышцам
2) Адреналин - связан с 1 пунктом
3) Метамбетанин - ну короче солдаты вермахта использовали. Он просто подавляет чувство усталости - поэтому можно работать на износ
Аноним 10/01/20 Птн 13:13:25 492145324
>>492141
а что вообще такое магнитное поле?
Аноним 10/01/20 Птн 13:15:45 492146325
>>492145
силовое поле, действующее на движущиеся электрические заряды и на тела, обладающие магнитным моментом, независимо от состояния их движения; магнитная составляющая электромагнитного поля.
Аноним 10/01/20 Птн 13:35:38 492148326
>>492146
>силовое поле
Что такое силовое? Бывает несиловое поле?
>действующее на движущиеся электрические заряды
А при помощи чего оно действует?
>магнитная составляющая электромагнитного поля.
Как это? Электромагнитное поле составное что ли? А может и все остальные поля составные?
Аноним 10/01/20 Птн 13:38:24 492149327
>>492148
> Бывает несиловое поле?
да
> Что такое силовое?
силовые магнитные поля обладают силой Лоренца в источниках
> А при помощи чего оно действует?
при помощи поля
> Электромагнитное поле составное что ли?
да
> А может и все остальные поля составные?
по большей части
Аноним 10/01/20 Птн 13:50:22 492150328
>>492149
>да
Это какое?
>обладают силой Лоренца
А что это за сила и кто ее переносит?
>при помощи поля
магия что ли? Поле действует при помощи поля?
>по большей части
А какие еще есть составные поля?
Аноним 10/01/20 Птн 13:59:18 492151329
>>492150
> Это какое?
https://elibrary.ru/item.asp?id=37638234
> А что это за сила
https://ru.wikipedia.org/wiki/%D0%A1%D0%B8%D0%BB%D0%B0_%D0%9B%D0%BE%D1%80%D0%B5%D0%BD%D1%86%D0%B0
> кто ее переносит?
поле
> Поле действует при помощи поля?
да, поле - это элементарный вид взаимодействия, он неделим
> А какие еще есть составные поля?
фермионные поля, бозонные поля
Аноним 10/01/20 Птн 14:02:05 492152330
>>492148
>Что такое силовое? Бывает несиловое поле?
Правильно называть потенциальное поле, но для интуитивного понимания силовые поля называют те поля, которые оказывают СИЛУ на что-либо. Вообще поле в физике это некая функция от координат или по-другому есть некоторая величина и она определена в каждой точке пространства(и времени). Например, поле температуры, поле скоростей, поле пшеницы. Так что поле вполне может быть несиловым.
>А при помощи чего оно действует?
Самого себя, но чтоб понять механизм действия, нужно углубляться в теормех.
>Электромагнитное поле составное что ли?
Нет, это единая величина. Но поскольку наше сознание мыслит категориями 3-мерного пространства, приходиться разделять поле на составляющие или компоненты. Когда переходим в 4-мерное пространство-время, все сводиться в простому и элегантному виду.
>А может и все остальные поля составные?
Зависит от того как мы определили поле.
Аноним 10/01/20 Птн 14:03:36 492153331
>>492151
>да, поле - это элементарный вид взаимодействия, он неделим
Что же тогда такое поле и откуда оно берется?
>бозонные поля
А из чего они состоят?
Аноним 10/01/20 Птн 14:05:57 492154332
>>492152
>Вообще поле в физике это некая функция от координат или по-другому есть некоторая величина и она определена в каждой точке пространства(и времени). Например, поле температуры, поле скоростей, поле пшеницы
А от каких координат берется функция для получения гравитационного поля?
>Но поскольку наше сознание мыслит категориями 3-мерного пространства
А как еще можно мыслить?
>Зависит от того как мы определили поле.
Ты же в самом начале написал что это такое. Как то еще можно его определить?
Аноним 10/01/20 Птн 14:07:32 492155333
Аноним 10/01/20 Птн 14:13:33 492156334
>>492155
>Поле в физике — форма материи: противопоставляется веществу.
А оно так же возникло в момент Большого Взрыва?
>тут сложно ответить
Так может бозонные поля не составные?
Аноним 10/01/20 Птн 14:14:27 492157335
>>492155
>Поле в физике — форма материи: противопоставляется веществу.
А выше ты пишешь, что есть фермионные поля. Так же так?
Аноним 10/01/20 Птн 14:17:52 492158336
>>492157
Не понял вопроса, объясните.
Аноним 10/01/20 Птн 14:19:52 492159337
>>492158
Ну ты в ответ на вопрос какие есть составные поля говоришь

>>492151
>фермионные поля, бозонные поля

Если есть фермионные поля, то как может быть, что

>>492155
>Поле противопоставляется веществу.
Аноним 10/01/20 Птн 14:20:38 492160338
>>492159
Так как фермионы это и есть вещество. Оно из фермонов состоит.
Аноним 10/01/20 Птн 14:23:59 492161339
>>492135
Спасибо, не принял это во внимание. Но я скорее интересовался с точки зрения локальности времени считая прочие условия одинаковыми. Нагуглил такой ответ https://physics.stackexchange.com/questions/126919/does-time-move-slower-at-the-equator но его не совсем понял, там выбрали ответ что разницы нет, а ниже кто-то приводит статью про ускорение хода атомных часов на вершине горы http://www.leapsecond.com/great2005/
Аноним 10/01/20 Птн 14:25:32 492162340
>>492161
>про ускорение хода атомных часов на вершине горы
Нет, лол. Физические процессы не ускоряются ни в какой из систем отсчета. Время ОТНОСИТЕЛЬНО, а не абсолютно.
Аноним 10/01/20 Птн 14:26:27 492164341
>>492154
>А от каких координат берется функция для получения гравитационного поля?
В ньютоновский от произвольной системы отсчета. В ОТО гравитационного поля нет, вместо него есть криволинейные координаты и произвольные от них тензоры метрики и кривизны. И тут систему координат можно выбирать произвольно, главное чтоб считать удобно было, что не всегда бывает.
>А как еще можно мыслить?
Категориями бесконечномерных пространств.
>Как то еще можно его определить?
Без задней мысли.
Определяем через величину, которую мы хотим определить. Допустим у нас говно растворенное в чане с говном, вводим в каждой точке чана значение плотности говна. Получаемая зависимость плотность говна от координат и есть наше поле. Поле можно определить и на площади, и на бесконечномерных пространствах. Все зависит от хотелок и скиллов.
Аноним 10/01/20 Птн 14:30:18 492167342
>>492164
>В ОТО гравитационного поля нет
Разве? А что же получается, фиксация гравитационных полн которая была совсем недавно показала неверность ОТО?
>Категориями бесконечномерных пространств.
А можно пример как это делать?
>Допустим у нас говно растворенное в чане с говном, вводим в каждой точке чана значение плотности говна.
Говно то откуда то взялось, а откуда взялись поля?

Аноним 10/01/20 Птн 14:33:50 492169343
>>492167
"Если же все тела в одной и той же точке пространства получают одинаковое ускорение, то это ускорение можно связать не со свойствами тел, а со свойствами самого пространства в этой точке.

Таким образом, описание гравитационного взаимодействия между телами можно свести к описанию пространства-времени, в котором двигаются тела. Эйнштейн предположил, что тела движутся по инерции, то есть так, что их ускорение в собственной системе отсчёта равно нулю. Траектории тел тогда будут геодезическими линиями
"
Аноним 10/01/20 Птн 14:34:13 492170344
>>492162
>Время ОТНОСИТЕЛЬНО
Относительно ЧЕГО? Вот смотри, движение - относительно.
Если ты и я - в двух машинах, и я двигаюсь, относительно тебя - вперёд, то ты, относительно меня - вдигаешься назад.

Также и с временем? Если одно время (время вблизи поверхности Земли) замедляется из-за гравитации,
ОТНОСИТЕЛЬНО другого времени (время на вершине горы, которая подальше от гравитационного центра),
то время на вершине горы - КАК-БЫ УСКОРЯЕТСЯ, ОТНОСИТЕЛЬНО времени, вблизи поверхности Земли.

Но это не так. Все мы знаем, что время может либо течь как текл, либо его течение может только замедлятся,
и время не может ускоряться, иначе это был бы путь к созданию машины времени и путешествие - в прошлое.
Аноним 10/01/20 Птн 14:35:28 492171345
>>492170
>...время может либо течь как текло, либо... бла-бла...
Аноним 10/01/20 Птн 14:37:17 492172346
>>492169
А как же?

Rμν - 1/2Rgμν + Λgμν = 8πG/c4 ⋅ Tμν
Аноним 10/01/20 Птн 14:37:43 492173347
>>492170
>Относительно ЧЕГО?
Относительно любой другой системы отсчета.
Аноним 10/01/20 Птн 14:39:35 492174348
>>492170
>Все мы знаем, что время может либо течь как текл, либо его течение может только замедлятся,
Время не ускоряется и не замедляется. Все физические процессы текут с одинаковой скоростью везде. Но для стороннего наблюдателя скорость их в какой-то другой системе отсчета может ускорятся или замедлятся. Это и есть относительность хода времени.
Аноним 10/01/20 Птн 14:40:21 492175349
>>492162
То есть ты хочешь сказать что типа чел сам по себе не заметит, что он двигается быстрее окружающего мира, если поднимется на гору, это заметит только наблюдатель у подножия, когда чел к нему спустится пожилым?

Вот тут пишут, что из-за этого у нас голова стареет быстрее тела. https://www.nist.gov/news-events/news/2010/09/nist-clock-experiment-demonstrates-your-head-older-your-feet

И еще: Before anyone rushes to lower elevations, though, the NIST scientists note that these effects are much too small for humans to perceive directly—adding up to approximately 90 billionths of a second over a 79-year lifetime. Вот тут не могу понять типа 1 секунда на 90 миллиардов или что фраза означает?
Аноним 10/01/20 Птн 14:42:58 492176350
>>492170
Таблетки выпей уже. Где бы ты не стоял, атом цезия в часах будет колебаться одинаково. Разница получается только при переходе в другую систему отсчета.
>>492172
Один из членов - тензор относящийся к кривизне пространства времени (вроде как разложенный тут на тензор и скаляр).
Аноним 10/01/20 Птн 14:43:40 492177351
>>492175
>То есть ты хочешь сказать что типа чел сам по себе не заметит, что он двигается быстрее окружающего мира, если поднимется на гору, это заметит только наблюдатель у подножия, когда чел к нему спустится пожилым?
Я хочу казать, что для этого чела часы этого чела будут идти как обычно и он не увидит, что стрелки начали крутиться как сумасшедшие ли наоборот встали.
Аноним 10/01/20 Птн 14:47:34 492180352
>>492176
>Один из членов - тензор относящийся к кривизне пространства времени
Хм, выходит что все дело в этом? Гравитация это просто геометрическая характеристика пространства-времени?


Аноним 10/01/20 Птн 14:48:22 492181353
>>492180
> Гравитация это просто геометрическая характеристика пространства-времени?
Это пишут в введение любого учебника по ОТО.
Аноним 10/01/20 Птн 14:49:18 492182354
>>492181
А не может ли быть так, что остальные поля это тоже геометрические характеристики пространства?
Аноним 10/01/20 Птн 14:50:29 492184355
>>492167
>Разве?
Конечно, это же говорится в принципе эквивалентности. Из-за особенной зависимости от системы координат, гравитацию нельзя строго назвать полем, поскольку это более сложная зависимость, чем просто функция от координат. Хотя если уж очень хочется, можно гравитацию назвать тензорным полем.
>А что же получается, фиксация гравитационных полн которая была совсем недавно показала неверность ОТО?
А как это получается? Именно из геометрического подхода в ОТО появляется решения соответствующие нашим гравитационным волнам.
>А можно пример как это делать?
Целый раздел математики есть - Функциональный анализ.
>Говно то откуда то взялось, а откуда взялись поля?
Боженька высрал. А поле же взялось при подсчета говна.
Аноним 10/01/20 Птн 14:53:07 492187356
>>492182
Может Этим занимается Гамильтонова механика.
Аноним 10/01/20 Птн 14:54:10 492188357
>>492184
>гравитацию нельзя строго назвать полем, поскольку это более сложная зависимость, чем просто функция от координат
А чем же тогда являются гравитационные волны? Волнам изменения геометрических характеристик пространства-времени?
>Боженька высрал. А поле же взялось при подсчета говна.
А как спаривается в этом вопросе теоретическая физика и космология? У космологов в их моделях используются полевая теория, так ведь? Они придумали откуда взялись поля?
Аноним 10/01/20 Птн 14:56:04 492190358
>>492187
>Может
Если так, то может ли фермионные поля быть таким же геометрическими характеристиками? Ведь из этого следует, что фермионы это ...ничто но сама геометрия пространства.
Аноним 10/01/20 Птн 14:58:44 492191359
220px-Quadrupol[...].gif (63Кб, 220x220)
220x220
Аноним 10/01/20 Птн 15:00:43 492192360
Аноним 10/01/20 Птн 15:02:05 492193361
image.png (32Кб, 630x355)
630x355
Аноним 10/01/20 Птн 15:02:59 492194362
>>492184
>Целый раздел математики есть - Функциональный анализ.
Там наверно очень умные люди работают.
Аноним 10/01/20 Птн 15:03:15 492195363
Аноним 10/01/20 Птн 15:06:34 492197364
Аноним 10/01/20 Птн 15:10:51 492199365
>>489769
Сознание может быть без мышления ("внутренней речи"). Любой практик медитации это знает. Ты сидишь (идешь, лежишь, стоишь и др.) и осознаёшь ощущения. Нет мыслей: Я есть, Я медитирую, Я хочу, Я наблюдаю - только наблюдение ощущений тела.
Я, речь от первого лица - языковые конструкции. Атеисты говорят СпасиБог, потому что так сложилось в русском языке, а не от веры в Бога.
А дети, кстати, говорят о себе в третьем лице. У них уже есть речь и еще нет Я.
Аноним 10/01/20 Птн 15:12:57 492200366
>>492199
>А дети, кстати, говорят о себе в третьем лице.
Ничего подобного. Мой сын когда заговорил сразу использовал первое лицо чтобы говорить о себе.
Аноним 10/01/20 Птн 15:15:32 492201367
>>492200
Ага. И первое что он сказал было: Я мыслю, а значит существую.
Аноним 10/01/20 Птн 15:16:31 492202368
>>492199
>А дети, кстати, говорят о себе в третьем лице
В японском тоже норма - вместо местоимения говорить своё имя.
Аноним 10/01/20 Птн 15:19:54 492203369
>>492201
>И первое что он сказал было
Он сказал Баба, потом уже мама, папа, потом начал еду говорит и игрушки, потом сказал "я пипи".
Аноним 10/01/20 Птн 15:28:38 492204370
Аноним 10/01/20 Птн 15:56:35 492205371
>>492201
Просто мать общается с детьми фразами "Мама сейчас будет готовить кушать, мама тебя оденет", а потом закономерно охуевает когда личинка повторяет стиль общения.
Аноним 10/01/20 Птн 23:03:08 492277372
Вучёные, а вот вы же считаете постоянно всякое.

Вот есть допустим задача, в которой изменение параметра, влияет на расчёт, а это изменение опять влияет на расчёт, а это изменение опять влияет на расчёт...
Как такое посчитать кроме как методом последовательных приближений?

И вот ещё одно, был у меня как-то экзамен, и вроде суть задачи была похожа на это: устройство выдаёт мощность время t, мощность при этом равномерно или нарастает от нуля до заданной величины, или падает с заданной величины до нуля.
Я посчитал это как взял среднее значение и умножил на время t.
А препод сказал неправильно и надо было через интеграл(экзамен не вышмат был а физика). Реально ли результат будет другой если посчитать как-то "правильно" или препод просто так доебался? И как это сделать, посчитать такое "правильно"?
Аноним 10/01/20 Птн 23:05:06 492278373
А какая там есть формула связанная с электричеством, что типа рост мощности подводимой, или выделяемой, приводит вроде как к росту температуры или выделению тепла в квадрате?
Или что-то такое похожее
Аноним 11/01/20 Суб 09:28:48 492343374
Аноним 11/01/20 Суб 10:16:59 492345375
>>492278
Блядь. Уж я думал, что закон Джоуля-Ленца даже двачеры из треда тупых знают.
Аноним 11/01/20 Суб 12:44:15 492356376
>>492343
>>492345
О, точно, оно.
Так вот, что за хуйня? Нет ли тут наёба закона сохранения энергии в квадрате?
Количество теплоты же равнозначно энергии.

Или тут смысл в том что P=IU?

Или в чём прикол.
Аноним 11/01/20 Суб 13:21:28 492362377
>>492356
>Или в чём прикол.
Прикол в том, что закон сохранения энергии устанавливает сохранение, сука, энергии. Вот если у тебя слева от знака равенства энергия, а справа - энергия в квадрате, тогда надо переживать.
А так чего неясного?
A=IUt, Q=IR2t
Аноним 11/01/20 Суб 13:22:43 492363378
Аноним 11/01/20 Суб 13:23:07 492364379
>>492362
>Прикол в том, что закон сохранения энергии устанавливает сохранение, сука, энергии
А он точно работает?
Аноним 11/01/20 Суб 13:23:44 492365380
SAM2398.jpg (120Кб, 1500x701)
1500x701
Анчоусы, помогите понять загадку мироздания.

На рис. 1 вы видите два контейнера из под курочки с лучком, запечённой в духовке для интересующихся могу предоставить рецепт, ежели кто от мамки съехал, а готовит не умеет. После эвакуации мяска в контейнерах осталась жижа (сиречь бульон), которая за пару дней это был спонтанный эксперимент, я не чухан, чесно!! в круглом контейнере приняла форму идеального круга. В то же время в прямоугольном контейнере форма образовавшегося объекта — аморфная.

Итак, вопрос вам, учёные: какие неведомые силы заставили соки юной цыпочки скукожиться в идеальную форму? Какого-то углубления в форме контейнера там нет.
Сам я подозреваю фрактальную природу сего феномена, при необходимости объясню подробнее.
Аноним 11/01/20 Суб 13:27:53 492366381
>>492365
> какие неведомые силы заставили соки юной цыпочки скукожиться в идеальную форму
Поверхностное натяжение
Аноним 11/01/20 Суб 13:28:49 492367382
Очень важный вопрос: нахуя в шапке "вопросов"? Убрать и все в треде встает на свои места.
Аноним 11/01/20 Суб 13:29:09 492368383
>>492365
>Итак, вопрос вам, учёные: какие неведомые силы заставили соки юной цыпочки скукожиться в идеальную форму
ФОрма дна емкости. Ты глазиком не видишь, но оно чутка вогнутое.
Аноним 11/01/20 Суб 13:29:57 492369384
>>492367
устойчивое выражение
Аноним 11/01/20 Суб 13:30:56 492370385
>>492368
Я всё там рассмотрел внимательнее, чем у юной цыпочки, нет там такого. Скорее дно именно что вогнутое (внутрь), а не выпуклое наружу.
>>492366
Так и думал!
Аноним 11/01/20 Суб 13:38:33 492371386
>>492277
> или препод просто так доебался
Зависит от точки зрения. Смысл задания не получить ответ (так как у тебя не какое-то там иследование иследование/опыт) а проверить понимаешь ли ты нахуя в физике интегралы. Если изменение равномерное то ответ был бы тотже (при не равномерном тоже мог совпасть, но там уже нюансы). Но ты не продемонстрировал владения интегралами, тоесть зафейлил задание.
Аноним 11/01/20 Суб 17:10:09 492470387
>>491494
АРАБОЛОИД, ыхых

аноны, есть картинка подходящая для такого названия ?
Аноним 11/01/20 Суб 18:15:13 492490388
>>492470
Лол, параболоид, конечно же.
11/01/20 Суб 20:45:42 492493389
11/01/20 Суб 20:46:17 492494390
>>492071
я был пьян
я и сейчас нетрезв
извините
Аноним 11/01/20 Суб 21:34:04 492495391
Допустим запилили ИИ (похуй что это невозможно). Разумные киборги выглядят как самостоятельные личности (пример Звездные Войны). Будут ли обладать схожими правами с людьми? В юридическом плане.
Аноним 11/01/20 Суб 21:53:02 492501392
>>492495
> Будут ли обладать схожими правами с людьми?
Чисто политическое решение. Могут да, могут нет.
Аноним 11/01/20 Суб 22:00:22 492502393
53290afe2a2e6af[...].jpg (24Кб, 613x533)
613x533
>>492501
Охуеный ответ. Вроде ответ дан, а вроде нихуя.
Аноним 11/01/20 Суб 22:16:56 492505394
>>492502
Ты херню спрашиваешь, ты можешь делать что хочешь. Правил нет
Аноним 11/01/20 Суб 22:41:02 492509395
Где найти настоящий тест на IQ?

Или как определить что он настоящий?

А то мне кажется много сайтов с левыми такими тестами, от балды составленными - которые не покажут настоящий уровень IQ у тестируемого.
Аноним 11/01/20 Суб 22:44:26 492510396
>>492509
Его нет, а все эти тесты вниманиеблядство. Ты или решаешь задачи которые перед тобой стоят и успешен, или идёшь нахуй.
Аноним 11/01/20 Суб 23:06:56 492515397
>>492505
Давай я тебя пошлю на хуй и дам ценный совет: не разевай пасть, если тебе нечего сказать.
Аноним 11/01/20 Суб 23:10:19 492517398
>>492277
Интегрально-дифференциальное исчисление было придумано именно для этого.
Аноним 11/01/20 Суб 23:28:20 492519399
>>492515
Хуй соси, даун тупой промытый.
Аноним 11/01/20 Суб 23:36:17 492521400
>>492366
В прямоугольном поверхностного не было?
Аноним 12/01/20 Вск 00:07:04 492522401
>>492519
Но даун ты сам и только что отсосал публично. Не расстраивайся, хуесос, в жизни все бывает.
Аноним 12/01/20 Вск 00:09:02 492523402
>>492522
Хренасе как тебе небомбит.
Аноним 12/01/20 Вск 02:00:30 492530403
>>492509
>Где найти настоящий тест на IQ?
Можешь сам его написать, он будет настолько же точен, как любой другой. IQ не имеет никакого отношения к науке и что измеряет этот "тест" никому неизвестно.
Аноним 12/01/20 Вск 02:06:39 492531404
>>492530
>никому неизвестно
Вообще-то это очень легко, этот тест будет показывает насколько хорошо ты можешь решать этот тест.
Аноним 12/01/20 Вск 03:51:55 492549405
Аноним 12/01/20 Вск 03:53:30 492550406
>>491620
Ой блядь, быдло, нахуй иди. Ты тупой что-ли? Если бы я не гуглил их, я бы просто не вкинул ссылку.
Аноним 12/01/20 Вск 03:57:29 492551407
Аноним 12/01/20 Вск 04:03:47 492554408
>>492550
Даунчик, нагуглить не значит разобраться.
Разбирайся дальше.
Аноним 12/01/20 Вск 04:06:23 492555409
>>492554
В чём разбираться-то? Меньше чем в них, нихуя не измеришь, а значит это кванты пространтва и времени, и пространство-время - дискретно.
Аноним 12/01/20 Вск 04:08:48 492556410
>>492555
Да да.

Давай так, скажи своими словами, что такое планковские величины.
Аноним 12/01/20 Вск 04:20:07 492559411
>>492555
>Меньше чем в них, нихуя не измеришь
Ага, особенно планковскую массу, 2x10^-5, прям вообще никак, весы с алика не потянут.
>а значит это кванты пространтва и времени
Ага, и ускорение дискретно, и нельзя ускорятся меньше чем на 5.5x10^51 м/с^2
И в электрической плитке нельзя выделяемую энергию подкрутить на шаг меньший чем 1.9x10^9 Дж, когда пельмешки варишь.

Ты мало того что не разобрался, ты их даже не просмотрел нормально.
Кванты блять.
Аноним 12/01/20 Вск 04:28:32 492561412
>>492556
Самая миллипиздрическая хуйня во Вселенной, меньше которой нихуя не доказано.
Аноним 12/01/20 Вск 04:40:19 492562413
>>492561
И как какой-то кукарек про >недоказано
Делает их квантами мироздания?
Аноним 12/01/20 Вск 04:41:02 492563414
>>492559
Я про массу и энергию нихуя не писал.
И хуле ты в производные планковские единицы полез?
https://ru.wikipedia.org/wiki/Планковские_единицы#Производные_единицы
Ты хоть сам разобрался в их определениях, да?
И ты, конечно же понимаешь, то это величина МАКСИМАЛЬНО-ВОЗМОЖНЫХ ЭНЕРГИИ и МАССЫ,
зажатой внутри космологической сингулярности - в планковский объём?
Больше этих значений - энергии-массы там быть не могло, в принципе,
так как планковскую энергию, имела Вселенная - в планковскую эпоху,
а масса больше планковской массы, зажатая в частице,
заставляет её коллапсировать в Чёрную Дыру (это нижний предел массы для черных дыр).

Про планковское ускорение там вообще редирект какой-то стоит, но судя по цифрам - это ебать какое ебическое ускорение.
Аноним 12/01/20 Вск 04:41:55 492564415
>>492561
Подумай ещё, очень жидкое и тупое определение.

Давай так, как они получились? В этом смысле что это такое, планковские величины?
Аноним 12/01/20 Вск 04:42:25 492565416
>>492562
А как недоказанность возможности измерений внутри них, не делает их - минимально-возможными квантами?
Аноним 12/01/20 Вск 04:49:53 492566417
>>492564
Они получились изначально, и специально для процесса структуризации континуума составлением из них.
Аноним 12/01/20 Вск 04:50:46 492567418
>>492566
Дискретного континуума.
Аноним 12/01/20 Вск 05:01:51 492569419
>>492565
Ты квазикристальный обоссаный даун? У него похожая логика.
Аноним 12/01/20 Вск 05:03:21 492570420
>>492569
Нет, я другой антон. У меня измерения, сжатие данных и планковские величины.
Аноним 12/01/20 Вск 05:04:31 492571421
>>492567
Но ведь пространство не дискретно. С чего ты решил, что математическая абстракция - планковская длина, означает, что пространство дискретно?
Аноним 12/01/20 Вск 05:05:07 492572422
>>492570
И чуток коммунизма ещё, только настоящего такого, реального, научно-обоснованного, а не фейкового.
Аноним 12/01/20 Вск 05:06:24 492574423
>>492571
Потому что если из этих отрезков сложить модель пространства, она, эта модель будет в достаточной степени - высокоточна.
Аноним 12/01/20 Вск 05:59:39 492605424
>>492603
Быстрее зашивайся... У тебя уже галлюцинации от кровопотери начались.
Аноним 12/01/20 Вск 06:14:41 492612425
>>492605
Кстати, если так вдуматься, и если реально,
планковская длина - наименьшая длина во Вселенной,
а планковское время - наименьший отрезок времени во Вселенной,
то исходя из этой статьи:
https://ru.wikipedia.org/wiki/Планковский_объём
вся Вселенная имеет ограниченный радиус и ограниченный объём:
>Радиус наблюдаемой вселенной оценивается в 2,7⋅10^61 планковских длин, а объём имеет порядок около 8,4⋅10^184 планковских объёмов.
а исходя из этой статьи:
https://ru.wikipedia.org/wiki/Планковское_время#Значение
ещё и ограниченное время существования:
>Время, прошедшее с момента Большого взрыва (4,3⋅10^17 с), примерно равняется 8⋅10^60 планковским временам.
при этом само существование её, началось опять же с планковского отрезка времени:
https://ru.wikipedia.org/wiki/История_Вселенной#Планковская_эпоха

Исхдя из вышеописанного, числа не очень большие, 8,4⋅10^184 - это единица с 185 нулями, всего.
С такими числами, можно было бы оперировать в обычном браузере, при помощи алгоритмов длинной арифметики,
как на этой вот страничке с JavaScript, позволяющем вычислять пиздатые кубы
и корни кубические из них: https://username1565.github.io/BigInteger.js/cube_root.html

Как видишь, значение куба может быть намного большим, нежели 10^184:
https://www.wolframalpha.com/input/?i=3756836213604256829445960352660150886037787357754893250960381869959956638123541871636786465736368038244374335857504137704265649741054521056490218634295322554729608213113984843303788234033417977
>3.7568362136042568294459603526601508860377873577548932... × 10^192

Так что, если пространство-время дискретно, то не удивляйся, фрагменты модели Вселенной - можно было бы рассчитать при помощи компов.
Аноним 12/01/20 Вск 06:16:27 492613426
>>492612
>не удивляйся, фрагменты модели Вселенной - можно было бы рассчитать при помощи компов.
И рассчитать так же быстро, как эти ебические кубы, пиздатющие.
Аноним 12/01/20 Вск 06:18:28 492614427
>>492613
>так же быстро
и высокоточно, безошибочно, просто и легко, без вычисления всякой хуиты ненужной и хранения её в памяти.
Аноним 12/01/20 Вск 06:19:12 492615428
>>492612
Уде лень читать, завтра посмотрю, но щаранее скажу что говновыводы, и глаз выцепил сразу пару обсёров подсознательно прямо уже, т.е. я их сейчас сказать не могу, но уже увидел их и знаю что они там есть.
Аноним 12/01/20 Вск 06:24:08 492618429
>>492612
Ахах, бля, хороший траллинг, мне понравилось.
Аноним 12/01/20 Вск 06:41:52 492626430
В чем профиты синестезии? Все на это дрочат прям пиздец, чуть ли не признаком сверхчеловека считают, но я понять не могу, что такого, в том, что ты можешь понюхать звук?
Аноним 12/01/20 Вск 07:05:17 492632431
>>492626
Отдельные счастливчики даже испытывают «цветной» оргазм.
Аноним 12/01/20 Вск 07:38:43 492637432
>>492626
> но я понять не могу
Так в этом и суть. Пойми.
Аноним 12/01/20 Вск 09:15:10 492650433
>>492612
Не путай модель и жизнь. Ну и модели дискретного пространства всё еще нет. Так что хватит срать тут.
Аноним 12/01/20 Вск 10:44:12 492665434
>>492650
Уже есть, давно причём. 20 лет как минимум есть.
Аноним 12/01/20 Вск 10:45:52 492666435
>>492665
>Уже есть, давно причём. 20 лет как минимум есть.
Как она называется?
Аноним 12/01/20 Вск 10:49:09 492667436
>>492612
> вся Вселенная имеет ограниченный радиус и ограниченный объём
Ты перепутал радиус наблюдаемой вселенной с радиусом вселенной.
Аноним 12/01/20 Вск 11:35:40 492679437
>>492666
Заебатость охуенная, вездесущая, глобальная, высокоточная, истинная, или просто - Бог.
Чтобы долго не объяснять быдлу - принципы её невъебенного и своевременного функционирования.
Аноним 12/01/20 Вск 11:44:25 492686438
>>492679
>или просто - Бог.
Религия что ли? А почему тогда --

>>492665
>20 лет как минимум есть.

Какая-то новая секта? Ты вступил в церковь дианетики?
Аноним 12/01/20 Вск 11:54:55 492696439
>>492686
Мань, заебатость же - вездесущая. Никакие секты не надо и церкви. В церквах просто дом Божий, там его повышенная концентрация. Но туда не обязательно переть, чтобы он объемлил всё.
Он и так, без походов в церкви, в состоянии объемлить всё и содержать всё - в Себе.
Аноним 12/01/20 Вск 11:55:33 492697440
>>492696
Через модель, лол.
Аноним 12/01/20 Вск 12:14:49 492700441
>>492667
Самая дальняя галактика - галактика UDFj-39546284, излучает свет,
который она излучала через 380 миллионов лет после Большого взрыва.
С тех пор свет доходит только до нас от этой галактики, и этот свет видно в телескопы.
https://ru.wikipedia.org/wiki/UDFj-39546284
А следовательно, размер Вселенной - конечен, а объём - ограничен.
Аноним 12/01/20 Вск 12:19:17 492701442
>>492700
Чем дальше смотришь, тем более ранние галактики и звёзды видишь, но у этого есть предел - возраст Вселенной,
который тоже конечен и ограничен, и не такими уж большими числами он исчисляется. Ага.
Аноним 12/01/20 Вск 12:31:36 492705443
>>492696
Давай помолимся, брат.
Аноним 12/01/20 Вск 12:32:20 492706444
>>492700
>А следовательно, размер Вселенной - конечен
Лол. У нас тут очередной философ.
Аноним 12/01/20 Вск 12:55:05 492708445
>>492700
>А следовательно, размер Вселенной - конечен, а объём - ограничен.
Если мы чего-то не видим, так как оно слишком далеко, то этого нет? Неплохо...
Аноним 12/01/20 Вск 13:36:14 492709446
>>492708
Ты понимаешь, что этого чего-то быть не может в принципе?
Аноним 12/01/20 Вск 13:37:13 492710447
>>492706
Ты понимаешь, хоть из чего это "следовательно" - следует?
Аноним 12/01/20 Вск 13:38:28 492711448
>>492710
Нет, не понимаю. Объясни ход своих рассуждений.
Аноним 12/01/20 Вск 13:48:52 492712449
>>492711
Если самый дальний объект, вышеуказанная галактика, была относительно молодой,
то возможный ещё более дальний объект - будет ещё моложе.
Но у "молодости" объекта этого есть предел - это возраст Вселенной.
Объект не может быть моложе планковского времени.
Значит, самый далёкий объект, который возможно было бы в принципе, узреть в телескопы -
должен бы иметь возраст не менее чем возраст Вселенной в планковскую эпоху,
и дальше этого объекта ты нихуя не увидишь в принципе.
Следовательно, радиус Вселенной, а значит и объём Вселенной - конечен.
Просто потому что свет оттуда доходит за время, равное возрасту Вселенной.
Аноним 12/01/20 Вск 13:50:58 492713450
>>492712
>Если самый дальний объект
А вот теперь вопрос. Что ты понимаешь под "дальний"? Какой способ измерения расстояния? Собственное расстояние? Сопутствующее расстояние? Расстояние по угловому диаметру? Расстояние по красному смещению?
Это очень важно понять, прежде чем мы продолжим.
Аноним 12/01/20 Вск 14:03:37 492714451
>>492713
Ну, блядь, реальное расстояние, которое пролетает свет за дохуя световых лет.
Это, походу - собственное расстояние, утверждённое и определённое по красному смещению.
Аноним 12/01/20 Вск 14:06:00 492715452
>>492714
Тащемта, тут: https://ru.wikipedia.org/wiki/UDFj-39546284
так и писано писарями этими космологическими:
>Однако, уже в декабре 2012 красное смещение UDFj-39546284 было уточнено — z=11,9, а расстояние до неё — не менее 13,42 млрд световых лет.
и модель Вселенной, эту писанину - одобряет, лол.
Аноним 12/01/20 Вск 14:10:28 492716453
>>492714
>реальное расстояние
Такого я не знаю в космологии.

>Это, походу - собственное расстояние

Собственное расстояние примерно соответствует расстоянию до места, где удалённый объект был бы в определённый момент космологического времени, измеренному с помощью длинного ряда линеек, протянутых от нашей позиции до позиции объекта в это время, и меняющемуся с течением времени в связи с расширением Вселенной.
Аноним 12/01/20 Вск 14:10:52 492717454
>>492715
>расстояние до неё — не менее 13,42 млрд световых лет.
Не указано какое именно.
Аноним 12/01/20 Вск 14:13:28 492718455
>>492716
Ну вот оно и есть. Галактика испустила свет, и летит себе в расширяющейся Вселенной, а свет летит охулиарды лет,
и потом в телескоп попадает, через линзы и зеркала - на матрицу.
Аноним 12/01/20 Вск 14:16:05 492719456
>>492718
Следующий пучок фотонов, потом долетает - немножко другое расстояние. Красное смещение считают между первым пучком и другим, определяют куда и как продвинулась галактика, где она была, с какой скоростью летит, откуда летит, и когда она образовалась, раз летит оттуда - туда, с такой-то скоростью.
Аноним 12/01/20 Вск 14:34:29 492721457
>>492718
Самое далекое расстояние, которые можно наблюдать находится от нас на расстоянии 46 миллиардов световых лет. Это поверхность условной сферы называющаяся поверхность последнего рассеивания. А время жизни Вселенной 13,8 миллиардов лет.
Если пользоваться твоими термінами то реально до поверхности последнего рассеивания сейчас 46 миллиардов световых лет. А те фотоны что мы регистрируем из этой области прилетели к нам с расстояния 13,4 миллиарда лет.
То что мы не видим дальше поверхности последнего рассеивания не значит, что там граница Вселенной, а лишь значит, что фотоны которые прилетают с границ поверхности это первые фотоны после того как Вселенная начала остывать и фотоны стали свободно по ней летать, вообщем это граница видимой НАМИ Вселенной.
Так вот, представь, ты смог телепортироваться к поверхность последнего рассеяния и решил лететь в ее направлении со скоростью света, ты огляделся и с удивлением увидел, что до поверхности последнего рассеяния 46 миллиардов световых лет. Ты решишь телепортироваться к ней и окажется, что ты переместился в пространстве, но до поверхности последнего рассеяния 46 миллиардов световых лет. Понимаешь почему так произошло?
Аноним 12/01/20 Вск 15:03:54 492722458
>>492712
Как ты относишся к тому что Земля находится в центре твоей конечной Вселенной?
Аноним 12/01/20 Вск 15:06:09 492723459
>>492721
>Самое далекое расстояние, которые можно наблюдать находится от нас на расстоянии 46 миллиардов световых лет.
Но его же, нельзя наблюдать?
>Это поверхность условной сферы
Сфера-то условная.
>А время жизни Вселенной 13,8 миллиардов лет.
Вот именно.
>То что мы не видим дальше поверхности последнего рассеивания не значит, что там граница Вселенной,
>а лишь значит, что фотоны которые прилетают с границ поверхности это первые фотоны
>после того как Вселенная начала остывать и фотоны стали свободно по ней летать,
>вообщем это граница видимой НАМИ Вселенной.
Тем не менее, возраст Вселенной - конечен, а значит - есть вполне определённый, какой-то предел.
То есть нельзя сказать, что если мы не видим дальше 13,8 млрд лет,
то Вселенной может быть больше 15-ти лярдов лет, и что-то там летало, но мы просто не видим - свет не дошёл.
>Так вот, представь, ты смог телепортироваться к поверхность последнего рассеяния
>и решил лететь в ее направлении со скоростью света,
>ты огляделся и с удивлением увидел, что до поверхности последнего рассеяния 46 миллиардов световых лет.
>Ты решишь телепортироваться к ней и окажется, что ты переместился в пространстве,
>но до поверхности последнего рассеяния 46 миллиардов световых лет.
>Понимаешь почему так произошло?
Не совсем понимаю. Я вообще не знаю откуда ты взял цифру 46 млрд. световых лет.
Но судя по этой статье: https://ru.wikipedia.org/wiki/Метагалактика#Размер
ты таки не пиздишь.
Полагаю, что несмотря на факт того, что Вселенной 13 с хуем лярдов лет,
она продолжает расширяться экспонециально, как в эпоху инфляции:
https://ru.wikipedia.org/wiki/История_Вселенной#Эпоха_раздувания_(инфляции)
то есть по такому вот закону:
https://ru.wikipedia.org/wiki/Инфляционная_модель_Вселенной#Инфляционное_расширение_на_ранних_стадиях_эволюции_Вселенной
и как видишь, там, в показателе степени - постоянная Хаббла, а значит, из-за показателя этого - расширение экспоненциально,
как на картинках, которые показывал Роджер Пенроуз, вот в этом видео: https://www.youtube.com/watch?v=la1IHh_9r9M
с 2:00 до 8:00.
И именно из-за экспоненциального расширения - такой пиздатый разрыв,
но сейчас, значение постоянной Хаббла упало по сравнению со значением в эпоху инфляции, и Вселенная расширяется медленнее,
но всё-равно по экспоненциальному закону. Поэтому, её диаметр больше, в световых годах, нежели её возраст.
А то, что ты описал, что расстояние к поверхности последнего рассеивания будет постоянным по 46 лярдов световых лет,
причём в любой точке Вселенной, я не пойму с чем это связано,
но мне кажется, что связано это с тем, что фотоны закольцованно двигаются в пиздатой Вселенной,
так как масса её совокупная - она пиздатюща, и эти галактики, всякие, в совокупности своей, гравитационные волны излучают,
они интерферируют, сливаются воедино, и всё это как-бы одна огромная чёрная дыра,
со всеми вытекающими, то есть там уже и гравитационное искривление континуума, и гравитационное линзирование,
ну ты понел.
Аноним 12/01/20 Вск 15:17:09 492724460
>>492723
>Но его же, нельзя наблюдать?
Можно. Только не в оптическом диапазоне. Это электромагнитные волны в микроволновом диапазоне. Это реликтовое излучение.

>Сфера-то условная.
Реальная поверхность последнего рассеивания выглядит как внутренняя поверхность сферы радиусом 46 миллиардов световых лет. А в центре Земля.

>Тем не менее, возраст Вселенной - конечен, а значит - есть вполне определённый, какой-то предел.
А если Вселенная родилась сразу бесконечной? Как тебе такая идея?

>То есть нельзя сказать, что если мы не видим дальше 13,8 млрд лет, то Вселенной может быть больше 15-ти лярдов лет

Эм, не понял твою мысль.

>Не совсем понимаю. Я вообще не знаю откуда ты взял цифру 46 млрд. световых лет.

Это расстояние до поверхности последнего рассеивания.
Аноним 12/01/20 Вск 15:27:53 492725461
>>492724
>>То есть нельзя сказать, что если мы не видим дальше 13,8 млрд лет, то Вселенной может быть больше 15-ти лярдов лет
>Эм, не понял твою мысль.
А ты выше прочитай и вместе:
>Тем не менее, возраст Вселенной - конечен, а значит - есть вполне определённый, какой-то предел.
>То есть нельзя сказать, что если мы не видим дальше 13,8 млрд лет, то Вселенной может быть больше 15-ти лярдов лет
Нутыпонел.

>А если Вселенная родилась сразу бесконечной? Как тебе такая идея?
Идея прикольная, но как она состыкуется с самим фактом наличия "поверхности последнего рассеяния", состоящей из реликтового излучения?
Исходя из природы излучения: https://ru.wikipedia.org/wiki/Реликтовое_излучение#Природа_излучения
появилось оно в результате рекомбинации электронов и протонов.
>Это случилось при температуре плазмы около 3000 К и примерном возрасте Вселенной 380 000 лет.
То есть, Вселенная не могла находиться ещё миллиард лет в таком состоянии.
Хотя... Поскольку время там было замедленным, из-за ебической гравитации,
вполне могло бы быть, так, что эти 380 000 лет могли бы протекать в течении каких-то условных килолярдов наших гигалет?
Аноним 12/01/20 Вск 15:35:56 492727462
>>492725
>возраст Вселенной - конечен, а значит - есть вполне определённый, какой-то предел.

Предел чего?

>Идея прикольная, но как она состыкуется с самим фактом наличия "поверхности последнего рассеяния", состоящей из реликтового излучения?

Очень легко. Скорость расширения пространства больше скорости света.

>Поскольку время там было замедленным

Почему ты так решил?
Аноним 12/01/20 Вск 15:54:15 492730463
>>492727
>Предел чего?
Максимально-возможный в принципе - предел расстояния, которое может преодолеть свет, с момента появления Вселенной.
>Очень легко. Скорость расширения пространства больше скорости света.
Тогда, соответствующее смещение должно бы быть ещё более красным, не? Это же как эффект Допплера!
>Почему ты так решил?
Потому что, "гравитационное замедление времени". Оно так и гуглится.
И вообще, там искривление всего континуума, поэтому загугли лучше это: https://ru.wikipedia.org/wiki/Гравитационная_линза
Аноним 12/01/20 Вск 16:00:39 492733464
>>492730
Даун с сыллками на википедию, ты опять тут мочёй воняешь?
Или это новый?
Ты в планковских величинах разбираешься?
Аноним 12/01/20 Вск 16:12:40 492735465
>>492730
>Тогда, соответствующее смещение должно бы быть ещё более красным, не? Это же как эффект Допплера!

Красное смещение, но природа у него несколько иная, это космологическая красное смещение оно вызывной тем, что грубо говоря когда электромагнитная волна летит в пространстве которое расширяется, то и длина волны расширяется. Если за время полёта фотона пространство расширилось в 3 раза, то и длина волны увеличивается в три раза.

>Потому что, "гравитационное замедление времени".

Со временем всегда все сложно, лол. Время не может "ускориться" или "замедлиться" абсолютно, оно может это сделать только относительно. Вообще лучше не лезь в тему времени, не ломай себе мозг окончательно.
Аноним 12/01/20 Вск 16:23:21 492739466
>>492730
>Максимально-возможный в принципе
До Земли. Еще раз ткну тебя, как уже двое сделали (один дважды) - у тебя в ЦЕНТРЕ ВСЕЛЕННОЙ ЗЕМЛЯ. Ты понимаешь, почему сложно поверить, что именно ничем непримечательная планета в обычной галактике ВРЯД ЛИ ЯВЛЯЕТСЯ ЦЕНТРОМ ВСЕЙ ВСЕЛЕННОЙ?
Берешь рандомную точку на границе видимой нами вселенной, умозрительно ставишь там Землю2 и получаешь внезапно НОВЫЙ ЦЕНТР ВСЕЛЕННОЙ. Теперь их два. Берешь точку еще на столько же парсеков дальше и ПОЛУЧАЕШЬ ТРЕТИЙ. Можешь продолжить или как умный человек сделать вывод, что пока у нас нет оснований полагать, что за границей ВИДИМОЙ вселенной нет еще сколько угодно парсек НЕВИДИМОЙ нами вселенной.
Написал я тебе правд не как умному человеку, но уж тут сорян бро, ты сам тупишь.
Аноним 12/01/20 Вск 16:27:32 492741467
>>492739
>До Земли. Еще раз ткну тебя, как уже двое сделали (один дважды) - у тебя в ЦЕНТРЕ ВСЕЛЕННОЙ ЗЕМЛЯ. Ты понимаешь, почему сложно поверить, что именно ничем непримечательная планета в обычной галактике ВРЯД ЛИ ЯВЛЯЕТСЯ ЦЕНТРОМ ВСЕЙ ВСЕЛЕННОЙ?

Ты зря... Он может и поверить.
Аноним 12/01/20 Вск 17:10:30 492749468
>>492733
Сразу нахуй иди. Ой бле, постой, а что же ты там мне приподнёс?
ПРОИЗВОДНЫЕ ПЛАНКОВСКИЕ ЕДИНИЦЫ, да?
Вот эти самые https://ru.wikipedia.org/wiki/Планковские_единицы#Производные_единицы
Вот с ними нахуй и иди, чмо хамовитое.
И не лезь, сюда, в серьёзный базар научный, пока папаша с вучёными тут базарит,
а то всем консорциумом, как облучим тебя бозонами нахуй, да прямо за щеку,
и будешь потом с флюсом ходить, как после отсоса недавнего.
Даун - это ты, уёбище лесное, а мочёй воняет, просто пушо ты не подмылся,
ты же только с хуя слез, куда ты срадостью, втихую, но в припрыжку, сразу и уёбываешь с треда.
Вот уебан, блядь, пукопостящий, пиздец нахуй, откуда ж ты высрался в моём няшном и ламповом - обоссае, а?

>>492735
>Красное смещение, но природа у него несколько иная,
>это космологическая красное смещение оно вызывной тем,
>что грубо говоря когда электромагнитная волна летит в пространстве которое расширяется,
>то и длина волны расширяется.
>Если за время полёта фотона пространство расширилось в 3 раза,
>то и длина волны увеличивается в три раза.
Ну вот, я об этом же. Если до момента рекомбинации электронов и протонов плазмы,
и образования реликтового излучения с его "поверхностью последнего рассеяния",
прошло энное количество миллиардов лет,
то всё это время Вселенная должна бы оставаться в таком положении,
и это - невозможно, при таких огромных давлениях и значениях констант, в том числе и значения постоянной Хаббла.
Иначе, пространство вынуждено было бы расширяться быстрее, не в 3 раза, скажем а в 30 раз,
и фотоны летели бы из более дальних точек, демонстрируя красное смещение в 10 раз превышающее, реальное.

>Со временем всегда все сложно, лол.
>Время не может "ускориться" или "замедлиться" абсолютно, оно может это сделать только относительно.
ОТНОСИТЕЛЬНО ЧЕГО?
>Вообще лучше не лезь в тему времени, не ломай себе мозг окончательно.
Ну вот и рассматривайте, сами, я вам подкинул своё видение, а вы там подумайте, теории свои расстройте,
откорректируйте формулы как надо, посмотрите чё да как там, и проверьте их на опыте,
авось и сами к высокоточной модели придёте,
в результате парочки простых коррекций, перекошенных издревле, лишь из-за человеческого фактора.

>>492739
>>492741
Чё-то бредни какие-то попёрли. Вы чё телепортироваться умеете? Какая нахуй Земля2?
Если Вселенная появилась 13,5 лярдов лет назад, то свет от самой дальней точки
доходит до Земли 13,5 лярдов лет, преодолевая расстояние в 13,5 лярдов световых лет.
Если ВНЕЗАПНО, переместиться туда, на ту границу,
то ты там нихуя не увидишь, потому что там УЖЕ НЕТ того, что было 13,5 лярдов лет назад.
Но, с той точки, происходило расширение ДРУГИХ частей Вселенной,
и от них УЖЕ ТУДА будет доходить свет, а ещё через 13,5 лярдов дойдёт свет и от Земли.
Но так как Вселенная расширяется экспоненциально (постоянную хаббла в показателе степени экспоненты видели, да?),
то там не 13,5 лярдов световых лет, а больше, 46 лярдов получается.
А теперь ты, >>492739 сам себя ткни, пикой в зад, мудло, блядское.
Аноним 12/01/20 Вск 17:20:35 492752469
>>492749
Не рвись, умный человек ты наш нет.
>>умозрительно ставишь там Землю2
>Вы чё телепортироваться умеете? Какая нахуй Земля2?
Ты даже в уме на такое не способен? Сирзли? Ты вообще млекопитающее?
>то ты там нихуя не увидишь,
Увижу примерно тоже самое, что и тут, галактики там, звезды, пустоту между ними. Это не нихуя, а очень даже дохуя, впрочем с хуями или без хуя - ты не понял нихуя.
>потому что там УЖЕ НЕТ того, что было 13,5 лярдов лет назад.
И, внезапно, оттуда ты тоже будешь видеть, что никакой земли еще нет в помине, а наша галактика еще даже формироваться не начала. Это и есть часть моего рассуждения, которое ты не смог осмыслить. От смены точки особо не меняется картина, с "Земли2" ты увидишь плюс-минус тоже самое.
>от них УЖЕ ТУДА будет доходить свет, а ещё через 13,5 лярдов дойдёт свет и от Земли.
Ты туповат. Это и есть мой аргумент, можешь мне его не возвращать, оставь себе, там поразмысли над ним.
>то там не 13,5 лярдов световых лет, а больше, 46 лярдов получается.
Я в курсе, найдешь у меня в посте 13,7 - выну хуй из-за твоей щеки и извинюсь.
>сам себя ткни, пикой в зад, мудло, блядское.
БУБУМЦ
Аноним 12/01/20 Вск 17:27:40 492754470
>>492749
Так забавно тупого дауна рвёт.
Аноним 12/01/20 Вск 17:30:50 492755471
>>492749
Ты бы лучше вместо того что бы ссылки на википедию давать
сам почитал эти статьи, разобрался.
Аноним 12/01/20 Вск 17:32:06 492756472
>>492749
Лoл зачем же ты так вдавил нoгoй этoгo червя прямo ебалoм oбратнo в землю?
Зачем же ты так прилoжил этoгo oмежнoгo инвалида с девствнным гoлoскoм?
Щас же ему срачеллo как прoстрелил, так oн зальет весь тред свoим бугуртoм и семенствoм.
OН же сейчас с ебалoм непoнимающегo дауна начнет в панике бегать и вертеться а пoтoм смачнo прoсрется пoд себя и сдoхнет.
Аноним 12/01/20 Вск 17:34:57 492757473
>>492754
Блядь, я просто не могy yже видеть эти попытки вайпа скyчного, семенящего yебка . Поскорее бы его yспыпили, как его спидознyю мамашy.
Аноним 12/01/20 Вск 18:07:08 492762474
>>492612>>492613>>492614
Было бы охуенно мгновенно перемещаться по заданной части модели Вселенной, не вычисляя всю Вселенную и её эволюцию.
Здесь, уже, отчасти, рассматривался этот вопрос: https://2ch.hk/sci/res/485179.html#492288
Аноним 12/01/20 Вск 23:59:46 492783475
>>492709
Хех... тогда у нас проблемы. Мы бы не наблюдали такую равномерную вселенную. А те самые далекие галактики и прочее говно, ведет себя так, будто вокруг них всё такое же как и у нас.
Аноним 13/01/20 Пнд 00:27:21 492784476
15700785037350.jpg (70Кб, 564x560)
564x560
Как вы читаете научпоп? Делаете конспекты, заметки, заучиваете отдельные фрагменты? У меня последнее время ощущение, что это немного юзлесс трата времени, так как по окончанию чтения того же Хокинга я только в общих чертах могу пересказать его "Краткую историю времени". Еще, например, когда читал "Расширенный фенотип" Докинза не смотря на то, что местами тоже туго шло и само чтение с перерывами заняло немало времени я не скажу, что я помню 100% содержания. Какие то ключевые мысли, отдельные факты, да, но в остальном усвоение содержания и близко не приближается к усвоению отдельного университетского курса. Возможно, у меня комплекс отличника, но в психач идти я не хочу. Из-за этого я бывает по несколько раз перечитываю отдельные фрагменты, потом еще раз главу повторяю главу, задаю себе вопросы проверочные, выделяя тезисы для себя. Хотелось бы читать больше, но такими темпами получается очень медленно и я никак не могу работать иначе, страшно что-то упускать. Особенно беспокоит когда ИРЛ я обсуждаю с кем-то прочитанную книгу, которую читал собеседник и я, и замечаю, что человек либо читал жопой, либо вообще нихуя не вынес из нее. Спрашивается, зачем же тогда ты читал ее, если она в тебя вошла и вышла. И я бы не хотел так.
Аноним 13/01/20 Пнд 00:39:32 492785477
>>492784
>Как вы читаете научпоп?
Никак. Нахуй его читать? Берешь учебник и читаешь.
Аноним 13/01/20 Пнд 00:39:45 492786478
>>492784
>Как вы читаете научпоп?
Зачем читать научпоп, когда можно читать учебники.
> что это немного юзлесс трата времени
Научпоп сам по себе абсолютная бесполезная трата времени.
Аноним 13/01/20 Пнд 01:16:48 492789479
>>492786
>когда можно читать учебники.
>>492785
>Берешь учебник и читаешь.
И какие же вы учебники прочитали?
Аноним 13/01/20 Пнд 01:29:39 492790480
>>492789
Пилю, хуле. Захотел понять что нужно есть/делать чтобы не толстеть зазря - взял учебник биохимии для вузов и без задней мысли прочитал, скипая самые зубодробительные реакции. Северина вроде.
Образование - АСУ.
Аноним 13/01/20 Пнд 01:40:54 492791481
>>492790
>взял учебник биохимии для вузов и без задней мысли прочитал
А ты поймешь что-нибудь из него без знания органической химии, цитологии и пр? Может надо перед этим прочитать учебник по химии и биологии?
Аноним 13/01/20 Пнд 01:46:30 492792482
>>492791
Ш К О Л А
К
О
Л
А
Ну да, хим-биол заканчивал, ну так и нехуй ходить в среднюю.
Аноним 13/01/20 Пнд 01:52:52 492794483
>>492789
>И какие же вы учебники прочитали?
Тебе что, весь список? Там десятки вузовских учебников.
Аноним 13/01/20 Пнд 02:21:26 492796484
>>492792
Школьного знания химии будет боюсь недостаточно чтобы понять университетский учебник биохимии.
Аноним 13/01/20 Пнд 02:46:18 492801485
Почему не делают процессоры CPU, которые бы могли разогреваться до 300 градусов (тепло бы отапливало дом), а размер чипа не делать размером с лист а0?

Получается у тебя дома CPU охуенный надолго и можно делиться теплом и расчётной силой.

Почему нет такого ещё?
Аноним 13/01/20 Пнд 03:02:30 492802486
>>492796
Ага, как и для использования знаков препинания? Цикл Кальвина для фотосинтеза мы в школе проходили/разбирали. На химии в лабораториях политеха по вечерам лабы делали (даже спирт гнали как-то). Не скажу, что учебник зашел легко, но понимать проблемы не было, особенно пропуская сложные реакции (опять же, экзамены сдавать не надо, так что и заучивать все ферменты необязательно).
>>492801
Эх, почему к ванне не приделают пропеллер и крылья, вот было бы охуенно летный лист заполнить, сделать ванну с пенкой, душ потянуть, перед отрывом с полосы и летать над дачами вокруг города! Почему нет такого еще?
Аноним 13/01/20 Пнд 03:19:39 492803487
>>492796
Да, забыл спросить, а для чтения учебника по макроэкономике для вузов - тоже недостаточно? Еще скажи, если книга понравилась, а перевод застрял на первой трети - школьного французского и за-компухтером-английского недостаточно чтобы начать читать Червя-непидора в оригинале?
Мне кажется, что ответ очень простой:
М О Т И В А Ц И Я
О
Т
И
В
А
Ц
И
Я
А вот если ее нет - даже пяти лет в профильном вузике будет недостаточно, чтобы учебник понять.
Аноним 13/01/20 Пнд 03:26:30 492804488
>>492723
> Сфера-то условная.
Это точно такой же ГС как у ЧД. Его ничто не пересечет, это самая настоящая граница Вселенной.
Аноним 13/01/20 Пнд 03:28:00 492805489
>>492804
Таблетки прими. Его можно хоть туда-сюда изъездить. Только ты опоздал, туда надо было выезжать немножк раньше.
Аноним 13/01/20 Пнд 03:32:47 492806490
>>492805
Только если умеешь перемещаться быстрее света.
Аноним 13/01/20 Пнд 03:37:50 492807491
>>492806
Еще раз - сейчас уже поздно, но если бы ты не сидел на дваче ночами, не прокрастинировал, а вышел пораньше (лучше сразу после периода инфляции), то сейчас бы мог там колесить туда-сюда сколько угодно. Дело не в самом расстоянии, а в ограничении V<=C + расширение. Ну или научиться гнуть пространство, тогда можно туда хоть сейчас скататься и убедиться, что нет там ГС, как у ЧД.
Аноним 13/01/20 Пнд 03:40:21 492808492
>>492807
У ЧД тоже нет ГС, если можно гнуть как угодно пространство.
Аноним 13/01/20 Пнд 03:47:21 492809493
Вопрос такой. Течение времени зависит, от скороси и массы объекта. На Земле время течет медленне чем на орбите нашей планеты. Еще медленне оно течет на Солнце. Крайний случай это черные дыры корые искажают течение времен , замедляя его пости до стазиса. Вопрос, что происходит со временем в великих пустотах(гиганских областях, где нету галактик)?
Если черные дыры это объекты где время с предельно возможным замедлением. Есть ли астрофизические объекты где течение времени сталкнется с таким же экстримальным ускорением?
Аноним 13/01/20 Пнд 03:52:44 492810494
142.png (240Кб, 472x369)
472x369
512.png (155Кб, 448x288)
448x288
Аноним 13/01/20 Пнд 03:55:36 492811495
>>492810
Поэтому ГС твоей Вселенной точно такой же как у ЧД. Ничто никогда его не пересечет.
Аноним 13/01/20 Пнд 03:55:45 492812496
>>492809
>Течение времени зависит, от скороси и массы объекта.
Иди нахуй.
>Крайний случай это черные дыры корые искажают течение времен , замедляя его пости до стазиса.
Зафиксирована смерть мозга, теперь только эвтаназия.
Аноним 13/01/20 Пнд 03:58:27 492813497
>>492811
Не ничто, а мы, не никогда, а вылетев сейчас. Чуешь разницу?
Аноним 13/01/20 Пнд 04:01:53 492814498
>>492813
Ну ты и наркоман, ЧД тоже можно покинуть ДО ее образования, лол. Кстати, а кто быстрее света может света перемещаться?
Аноним 13/01/20 Пнд 04:10:31 492816499
>>492814
Ты сразу и толстый, и невменяемый. Ну и комбо. Дело не в образовании, а в расширении пространства.
Хорошо, пример для полного дауна:
Есть поезд, который начал движение с ускорением вдоль платформы, есть ты в первом вагоне, есть мамка, которая тебе машет снаружи. Ты начинаешь идти вдоль вагона, твоя максимальная скорость 5 км/ч, пока поезд движется со скоростью менее 5 км/ч ты спокойно продолжаешь махать мамке, но постепенно она начинает удаляться, и ты не можешь оставаться вровень и продолжать махать, мало того, спустя непродолжительное время, ты даже перестаешь ее видеть в окнах. Из чего следует глубокомысленный вывод, что мамка только что перестала существовать, да и платформа тоже заодно. Мало того, следом еще более гениальный: поезд едущий в противоположном направлении невозможен!
Вот он блидинг ейдж науки-то!
Аноним 13/01/20 Пнд 04:13:28 492817500
>>492812
Ты спорешь с Энштейном? Типа парадокс близнецов, все дела.
Это факт черная дыра это объект где время течет очень медленно. Зажженая спичка в окресностях черной дыры может прогореть время существования нашей галактики. Зависит как близко от сингулярности ты будешь. Для спички пройдет всего пара секунд.
Аноним 13/01/20 Пнд 04:14:12 492818501
>>492816
> Из чего следует глубокомысленный вывод, что мамка только что перестала существовать
Если она больше недостижима, то так и есть. Ты действительно не согласен с тем, что нас окружает ГС на расстоянии 46 млрд св лет?
Аноним 13/01/20 Пнд 04:14:56 492819502
23
fxd
Аноним 13/01/20 Пнд 04:15:24 492820503
Ан ет все правильно
Аноним 13/01/20 Пнд 04:19:08 492821504
1806.png (266Кб, 564x446)
564x446
>>492817
>Ты спорешь с Энштейном?
Нет, с ним я согласен, окромя его неприятия КМ. Я спорю с дебилом с двача, у которого "время замедляется". Эйнштейн бы тебе в рот насрал, если бы не был культурным немвреем, а был бы двачером.
>черная дыра это объект где время течет очень медленно
Ебанутый.
Ты хоть раз задумывался, к чему в в теории относительности слово "относительность"?
>>492818
Ты нашел способ, как стирать людей из реальности. Пикрил.
Аноним 13/01/20 Пнд 04:21:02 492822505
232.jpg (34Кб, 600x630)
600x630
>>492816
> поезд едущий в противоположном направлении невозможен!
Все же Отнасительна!
>>492821
Это способ уюрать из реальности что угодно, просто отправить это за ГС и не важно ГС это ЧД или нашей Вселенной, по природе они одинаковы -- из них не выбраться
Аноним 13/01/20 Пнд 04:27:13 492823506
>>492822
Тебя не смущает даже, что ГС у ЧД снаружи, а то о чем ты говоришь - мы в внутри? Это нам, дебил, туда не выбраться. Причем по разным причинам. Опять же, останови расширение - и вуаля, снова можно выбраться, освой изгиб пространства - снова выбраться, а самое главное, возьми точку близкую к этой твой ГС, и окажется, что то место, куда не можешь попасть ты - оттуда спокойно достижимо. Например сидящий в вагоне 13 Ерохин успеет высунуть из окна свой хуй и по губам мамки провести, которую ты уже преждевременно отправил в великое ничто.
Аноним 13/01/20 Пнд 04:33:47 492824507
>>492821
А что оно делает относительно земного. Замедляется.
Относительность? Ко многому. Искажение пространства это искажение течение времени. Можно сказать, что масса объекта искажает пространство, можно сказать течение времени. Оба утверждение верны.
Аноним 13/01/20 Пнд 04:39:13 492825508
>>492824
>относительно земного
Вот теперь похоже на то, что ты что-то понимаешь. Только не пространства или времени, а пространства-времени. (x,y,z,)(t), одно без другого никак. Но ведь в следующем посте ты опять скатишься на ускорение-замедление времени, без второй системы отсчета, не так ли?
Переформулируешь свой изначальный вопрос правильно?
Аноним 13/01/20 Пнд 04:41:52 492826509
>>492824
Из ЧД не выбраться, из Вселенной не выбраться. Что не так?
Аноним 13/01/20 Пнд 04:42:56 492827510
>>492802
>Цикл Кальвина для фотосинтеза мы в школе проходили/разбирали.
В самых общих чертах, так что не нужно рассказывать сказки, что ты понял университетский курс биохимии просто прочитав учебник.
Аноним 13/01/20 Пнд 04:44:33 492828511
>>492826
Ерохин по губам твоей мамки стучит, вот что не так. Он выбрался туда, за край, из 13 вагона. Как объяснишь?
Аноним 13/01/20 Пнд 04:47:57 492829512
>>492809
>На Земле время течет медленне чем на орбите нашей планеты. Еще медленне оно течет на Солнце. Крайний случай это черные дыры корые искажают течение времен , замедляя его пости до стазиса.

Время везде течет одинаково и на Земле и на орбите и на Солнце. Разница в ходе течения времени относительная. Т.е. относительно тебя время на орбите течет медленно, а относительно Солнца оно течет быстро. Но при этом внутри всех этих трех систем отсчета время течет с одинаковой скоростью.
Аноним 13/01/20 Пнд 04:50:48 492830513
>>492828
Вагоны в одном составе, я не вижу где тут должна быть проблема.
Аноним 13/01/20 Пнд 04:51:14 492831514
>>492827
Ну все, пришел двачер и мне рассказал, что и как я проходил, что и как я понял. Теперь все встало на свои места! Как я без твоего анализа моей жизни исходя из одного поста жил?
Аноним 13/01/20 Пнд 04:53:43 492832515
>>492825
Почему если о гравитации говорят на примере прастранства, я не могу сформлировать на примере времени?
Черные дыры искривляют пространство и время. Соответственно что делают великие пустоты? И вообще умесно ли этотсравнение? Только не надо мне втирать об темных энергиях вопрос не об этом.
Аноним 13/01/20 Пнд 04:59:31 492833516
>>492829
Хороше берем Землю, нейтроной звезду и центр одной из великих пустот.
Аноним 13/01/20 Пнд 04:59:45 492834517
>>492830
В том, что если ты не смог достать - это не значит, что никто не смог. Так что перенеси себя на любое место внутри этой сферы и вокруг будет такая же сфера уже оттуда. Т.е. ты можешь отлететь на 1 световой год от земли и будет область пространства, которая находится за пределами этого "ГС" для Земли, но внутри - для тебя. Ты будешь видеть свет, который никогда не дойдет до Земли и вице верса.
Аноним 13/01/20 Пнд 05:01:23 492835518
>>492834
Как это противоречит мною сказанному?
Аноним 13/01/20 Пнд 05:06:57 492836519
>>492832
>Соответственно что делают великие пустоты? Только не надо мне втирать об темных энергиях вопрос не об этом.
Ничего. Там нет массы, искривляющей пространство-время, так что пока ты берешь точки внутри этой области - ты будешь наблюдать, что 9 192 631 770 периодов излучения, соответствующего переходу между двумя сверхтонкими уровнями основного состояния атома цезия-133 где-то в той же области, произойдут за секунду по твоим часам.
Аноним 13/01/20 Пнд 05:10:16 492837520
>>492835
>Максимально-возможный в принципе - предел расстояния, которое может преодолеть свет, с момента появления Вселенной.
Отлетел на 1 световой год -
Аноним 13/01/20 Пнд 05:13:25 492838521
>>492837
Сори, я многим позже вклинился в нить. Мой тезис только в том, что ГС Вселенной по сути такой же ГС как у ЧД, пускай и природа их возникновения различна.
Аноним 13/01/20 Пнд 05:14:15 492839522
>>492837
>Это точно такой же ГС как у ЧД. Его ничто не пересечет, это самая настоящая граница Вселенной.
Сидишь на расстоянии световой год от Земли - смотришь, как кто-то его пролетел.
>Как это противоречит мною сказанному?
Да, не, все норм.
Аноним 13/01/20 Пнд 05:20:53 492840523
>>492838
В том плане, что со скоростью света уже не вылететь "по сути такой же"? Ну так разница простая. Со скоростью света даже не влететь. Ну и наблюдаемый - 46, а выпусти сигнал сейчас, он за другое время (расстояние) достигнет новой "последней точки", дальше которой его "съест" красное смещение.
Аноним 13/01/20 Пнд 05:23:14 492841524
>>492840
"Граница Вселенной", ГС именно что съедает все сигналы, даже за бесконечное время они не пересекут эту границу. Более того, эта граница становится все ближе и ближе.
Аноним 13/01/20 Пнд 05:25:34 492842525
>>492836
Ты меня троллишь.
Тебе должно быть совесно.
Аноним 13/01/20 Пнд 05:25:36 492843526
>>492841
Чем же это граница, если съели твои сигналы, а от соседней звезды - не съели? Они там данные обработали и через несколько лет тебе прислали фотки из-за твоего края.
Аноним 13/01/20 Пнд 05:27:30 492844527
>>492843
Забеал, за 46 млрд св лет мне ничего не пришлют.
Аноним 13/01/20 Пнд 05:29:25 492845528
>>492844
От галактики на расстоянии 1 млрд св. лет через миллиард + n лет пришла тебе фотка, отправленная сегодня. На ней область пространства за пределами 46 млрд св. лет от земли. И прифотошопленный к твоему рту хуй.
Аноним 13/01/20 Пнд 05:31:35 492846529
>>492845
> отправленная сегодня
Миллирад +n лет назад.
Аноним 13/01/20 Пнд 05:32:09 492847530
Аноним 13/01/20 Пнд 05:33:16 492848531
>>492847
> сегодня
Ага, из сейчас в сейчас из отсюда сюда же.
Аноним 13/01/20 Пнд 05:33:37 492849532
karta-reliktovo[...].jpg (293Кб, 1600x800)
1600x800
>>492811
>Поэтому ГС твоей Вселенной точно такой же как у ЧД. Ничто никогда его не пересечет.

Это не совсем горизонт событий как у ЧД. Поверхность последнего рассеяния это не некий барьер и не некая граница.
Это довольно условная штука, ведь у каждой точке Вселенной есть поверхность последнего рассеяния. До этой поверхности 46 миллиардов световых лет, если ты перенесешься с Земли на расстояние 47 миллиардов световых лет, то ты окажешься обычной тебе Вселенной и увидишь, что до поверхности последнего рассеяния 46 миллиардов световых лет.

У микроволнового фона реликтового излучения которое приходит с поверхности последнего рассеяния есть неоднородности температуры, в каких то местах оно она доли градуса теплее, а из каких-то на доли градуса холоднее. Пикрелейтед это карта этого микроволнового фона. Считается, что вот эти флуктуации температуры определены вариациями распределения материи сложившимися во Вселенной на момент возникновения реликтового излучения, карта юной Вселенной.

Поверхность последнего рассеяния это не барьер который невозможно пересечь, аналогия с горизонтом черной дыры неверна. Это всего-лишь пространственно-временная граница, которая, сорри за тавтологию, ограничивает объем Вселенной, доступный наблюдениям в электромагнитных волнах из-за того, что Вселенной 13, 8 миллиардов лет и скорость света конечна. Если бы ты перенесся во времени допустим на 13 миллиардов лет назад, то увидел бы, что до поверхности последнего рассеяния всего пару миллиардов световых лет (800 миллионов лет + некая переменная из-а того что само пространство расширяется).
Аноним 13/01/20 Пнд 05:36:04 492850533
>>492848
Ты читай глазами:
Из галактики на расстоянии 1 млрд св. лет
сегодня сделали и послали фотку
через миллиард + n (на расширение) лет
ты ее получил
там область пространства, которая сегодня (в день отправки)
находится за пределами сферы радиусом 46 млрд св лет от Земли.
Аноним 13/01/20 Пнд 05:36:16 492851534
>>492849
> это не некий барьер и не некая граница
ГС ЧД это тоже условная (относительная) штука. Падая в очень большую ЧД можно и не заметить как пересекаешь ГС.
> Это всего-лишь пространственно-временная граница, которая, сорри за тавтологию, ограничивает объем Вселенной, доступный наблюдениям в электромагнитных волнах
Это и есть горизонт событий!!!
Аноним 13/01/20 Пнд 05:36:17 492852535
>>492841
Границы вселенной нету. Есть лимит на передачу информации во вселенной. Это как горизонт. Ты видешь горизонт. Куда бы ты не пошел ты будешь видеть гаризонт. Но гаризонта нету. Это всего лишь лимит в следствии кривизны земли. Граница обусловленна скоростью света. Которая канстанта на скорость передачи информации. Скорость фатона обуславливается свойством прастранство временного континиума, а не его максимальной скорость.Фатон такой быстрый, что он уперся в стену.
Аноним 13/01/20 Пнд 05:36:35 492853536
>>492831
>Ну все, пришел двачер и мне рассказал, что и как я проходил
Все двачеры учились в школе и помнят, что фотосинтез, цикл трикарбоновых кислот и пр. в школьном учебнике преподается в самых общих чертах и этих знаний явно недостаточно чтобы просто взять университетский учебник биохимии, прочитать его и схожу все верно понять.
Аноним 13/01/20 Пнд 05:37:29 492854537
>>492833
>Хороше берем Землю, нейтроной звезду и центр одной из великих пустот.

Взяли. И что с этим делаем?
Аноним 13/01/20 Пнд 05:37:42 492855538
>>492852
Границей я и называю горизонт, потому что горизонт по сути граница.
Аноним 13/01/20 Пнд 05:37:58 492856539
>>492851
>ГС ЧД это тоже условная (относительная) штука.
Ебать ты даун. Хоть слева зайди, хоть справа, а горизонт событий чд будет (для невращающейся) на одном и том же расстоянии. Отойди на шаг - останется там же.
Аноним 13/01/20 Пнд 05:39:07 492857540
>>492853
>>492792
Читай под спойлером. Потом грусти, что ты учился в школе, где не было практических лабораторных работ по генетике человека.
Аноним 13/01/20 Пнд 05:39:26 492858541
>>492856
Заглянуть за него ты не сможешь как с "границей" Вселенной.
Аноним 13/01/20 Пнд 05:41:48 492859542
>>492850
> которая сегодня (в день отправки)
Абсолютного времени не существует. "Сегодня" это координата пространства-времени. Мне отправили фотку за миллиард км, а не сегодня.
Аноним 13/01/20 Пнд 05:42:07 492860543
>>492858
>как с "границей" Вселенной.
Сейчас и отсюда? Не смогу. Пришелец из соседней галактики? За нашу - сможет. Может сделать фотку, прислать.
Аноним 13/01/20 Пнд 05:42:19 492861544
>>492851
>ГС ЧД это тоже условная (относительная) штука.
Вообще-то это самая не условная штука во Вселенной.
У нее есть координаты в пространстве, в отличие от поверхности последнего рассеяния.
Аноним 13/01/20 Пнд 05:43:03 492862545
>>492857
>в школе, где не было практических лабораторных работ по генетике человека.
Лол, не знал, что в России есть такие школы. Сорян.
Аноним 13/01/20 Пнд 05:43:58 492863546
>>492859
Сегодня по нашим часам. 13.01.2020 от рх. Мы узнали об этом получив сигнал (через миллиард с чем-то лет) и зная расстояние. Ты виляешь.
Аноним 13/01/20 Пнд 05:44:07 492864547
>>492861
> У нее есть координаты в пространстве, в отличие от поверхности последнего рассеяния.
Разве нам не известны границы нашей Вселенной?
Аноним 13/01/20 Пнд 05:46:01 492865548
>>492863
Лол, но ведь на фото будет ровно то же, что я вижу в телескоп.
Аноним 13/01/20 Пнд 05:46:07 492866549
>>492864
Границы нашей бесконечной однородной вселенной? Ну хууууй знает
Аноним 13/01/20 Пнд 05:46:39 492867550
>>492864
>Разве нам не известны границы нашей Вселенной?
Конечно нет. Ты думаешь, что поверхность последнего рассеяния это граница Вселенной? Это всего временная характеристика для фотонов реликтового излучения. Это просто самые старые фотоны в нашей Вселенной, вот и все.
Аноним 13/01/20 Пнд 05:46:59 492868551
>>492866
Нет, границы той наблюдаемой Вселенной, которая с большой буквы пишется.
Аноним 13/01/20 Пнд 05:47:03 492869552
>>492865
На фото будет то, что ты никогда не увидишь в телескоп. Галактики, звезды, свет от которых не дойдет никогда до тебя, как и это объяснение.
Аноним 13/01/20 Пнд 05:47:58 492870553
>>492867
Я отождествляю эти два понятия, так они обладают схожими характеристиками.
Аноним 13/01/20 Пнд 05:48:39 492871554
>>492869
Проиграл. Такое возможно только, если мне пришлют фото не за миллиард лет, а в одно мгновенье в прошлое.
Аноним 13/01/20 Пнд 05:51:41 492872555
>>492870
>они обладают схожими характеристиками.

Это разные штуки. Горизонт событий ЧД реален, у него есть координаты в пространстве и его можно пересечь только в одном направлении. Поверхность последнего рассеяния не имеет координат в пространстве и ты никогда не сможешь к ней приблизиться, летя к этой поверхности ты увидишь, что расстояние до нее не уменьшается.
Аноним 13/01/20 Пнд 05:52:18 492873556
>>492871
Вот-вот, никогда не дойдет до тебя. Они на млрд св лет ближе к этой границе, значит они видят, что было в области в 47 млрд. св. лет от тебя (в 46 от них), и не видят (с другой стороны сферы) на 1 млрд. св. лет (47 от них), но который еще видишь ты.
Сфоткали, прислали, ты сфоткал и прислал. Через миллиард с чем-то лет полюбовались.
Аноним 13/01/20 Пнд 05:54:14 492874557
>>492868
>наблюдаемой Вселенной
Это не вселенная, а лишь ее область, которую мы сейчас видим. Ну вот такой калека фотон, что не поспел за расширением.
>которая с большой буквы пишется
Ну хоть один аргумент в споре нашелся, когда остальные иссякли.
Аноним 13/01/20 Пнд 05:56:33 492875558
Кто перекатит?
Аноним 13/01/20 Пнд 05:57:34 492876559
Тред настолько расширился, что его фотоны уже не долетают до нулевой.
Аноним 13/01/20 Пнд 05:59:51 492877560
Безымянный.jpg (44Кб, 1369x655)
1369x655
>>492873
> Сфоткали, прислали, ты сфоткал и прислал. Через миллиард с чем-то лет полюбовались.
Это невозможно. Мне могут прислать только то, что я и так могу увидеть.
Аноним 13/01/20 Пнд 06:00:32 492878561
Перекатывайте, расскажу еще о "границе" во Вселенной.
Аноним 13/01/20 Пнд 06:02:07 492879562
0017.png (13Кб, 698x432)
698x432
>>492877
>Это невозможно. Мне могут прислать только то, что я и так могу увидеть.
Хуя се, т.е. в 1 млрд. св. лет скорость света не равна С? Или расширение (красное смещение) оттуда другое?
Аноним 13/01/20 Пнд 06:04:29 492880563
>>492874
Я не говорю про гипотетическую, воображаемую вставленную которую которая сродни бредням религоблядей никто никогда не видел и не увидит. Я говорю только про наблюдаемую Вселенную.
>>492872
Относительные координаты у этого последнего рассеяния есть. И потому я никогда не доберусь до него это и есть граница. Это буквально граница моих возможностей.
Аноним 13/01/20 Пнд 06:04:49 492881564
>>492878
Пусть это говно тут останется, не боись, еще пару часов точно не смоет, лол.
Аноним 13/01/20 Пнд 06:06:34 492882565
>>492879
Так расширение и не даст передать мне недостижимую инфу. Все уйдет в красный спектр и далее.
Аноним 13/01/20 Пнд 06:07:54 492884566
Перекат: https://2ch.hk/sci/res/492883.html
Перекат: >>492883 (OP)
Перекат: https://2ch.hk/sci/res/492883.html
Перекат: >>492883 (OP)
Перекат: https://2ch.hk/sci/res/492883.html
Перекат: >>492883 (OP)
Перекат: https://2ch.hk/sci/res/492883.html
Перекат: >>492883 (OP)
Перекат: https://2ch.hk/sci/res/492883.html
Перекат: >>492883 (OP)
Перекат: https://2ch.hk/sci/res/492883.html
Перекат: >>492883 (OP)
Перекат: https://2ch.hk/sci/res/492883.html
Перекат: >>492883 (OP)
Перекат: https://2ch.hk/sci/res/492883.html
Перекат: >>492883 (OP)
Перекат: https://2ch.hk/sci/res/492883.html
Перекат: >>492883 (OP)
Перекат: https://2ch.hk/sci/res/492883.html
Перекат: >>492883 (OP)
Перекат: https://2ch.hk/sci/res/492883.html
Перекат: >>492883 (OP)
Перекат: https://2ch.hk/sci/res/492883.html
Перекат: >>492883 (OP)
Перекат: https://2ch.hk/sci/res/492883.html
Перекат: >>492883 (OP)
Перекат: https://2ch.hk/sci/res/492883.html
Перекат: >>492883 (OP)
Перекат: https://2ch.hk/sci/res/492883.html
Перекат: >>492883 (OP)
Аноним 13/01/20 Пнд 06:11:07 492889567
>>492880
>Относительные координаты у этого последнего рассеяния есть

Лол, они есть для каждой точки в пространстве. И от каждой точки в пространстве до поверхности последнего растения сейчас 46 миллиардов световых лет. А 10 миллиардлв лет было гораздо меньше чем 46 миллиардов световых лет, а через 10 миллиардов лет будет гораздо больше чем 46 миллиардов световых лет.
Настройки X
Ответить в тред X
15000 [S]
Макс объем: 40Mб, макс кол-во файлов: 4
Кликни/брось файл/ctrl-v
Стикеры X
Избранное / Топ тредов